Shared Flashcard Set

Details

NAVLE
Boards questions
153
Veterinary Medicine
Graduate
09/12/2012

Additional Veterinary Medicine Flashcards

 


 

Cards

Term
A 4-year old quarterhorse presents with a 3 week history of diffuse patchy alopecia on the ventral midline and face, including a "bulls-eye" lesion in the center of the forehead. There is a mild conjunctivitis and uveitis OU (both eyes). What is the most appropriate treatment? A - Trichlorfon drench, antibiotic ophthalmic ointment TID B - Topical DMSO, Oxybendazole q. 8 weeks C - Ivermectin now, repeat in 4 months D - Corticosteriods, benzimidazole-piperazine q. 12 weeks E - Do nothing
Definition
Answer: C - Ivermectin now & in 4 wks The treatment of choice for Onchocerca spp microfilaria is an avermection-family drug, with retreatment at 4 month intervals.Treated horses show marked improvement. Although adult filaria are not killed by avermectins, in practice most horses are free of disease within 6-12 months. Adult filaria live in calcified nodules in the ligamentum nuchae and produce microfilaria that migrate to the ventral midline, face, neck, and chest. The resulting dermatitis is thought to be a hypersensitivity to microfilarial antigens. Accumulation of microfilaria in the eye may also lead to conjunctivitis and uveitis. Note that Culicoides spp (no-see-ums, biting midges) carry the onchocerca microfilariae, but do not cause the problems listed above themselves. Instead, biting midges cause sweet itch, a very pruritic warm weather dermatitis that is also secondary to hypersensitivity. Doing nothing is a poor choice in this symptomatic animal. Finding microfilaria in skin biopsies of asymptomatic horses is a common incidental finding. Because of this, some references debate the role of onchocera in the pathogenesis. It seems likely that variations in immune response play a role in severity of disease presentation. In humans, Onchocerciasis is a well known cause of hypersensitivity-related superficial keratitis (river blindness) and dermatitis.
Term
An 11 year old neutered male Doberman pinscher is presented with a 2 week history of limping on the left fore. The medial digit is swollen, the nail is deviated laterally. Ulceration and proliferation are present in the ventral nail bed.

Cytology of the mass is inconclusive. Chest radiographs are clear. Following amputation of the digit, histopathology indicates that the mass is a malignant melanoma.

What is the prognosis for this case?

A - Excellent
B - Good
C - Fair
D - Poor
E - Cannot say
Definition
Answer: D - Poor
The prognosis is poor. Melanoma of the digit is an aggressive, infiltrative tumor, with substantial metastatic potential and poorer prognosis. Median survival with resection is 12 months, with a 30% local recurrence rate. Melanomas are not thought to be responsive to radiotherapy, and the efficacy of chemotherapy for subungual melanomas is unclear.

Remember that a malignant melanoma can appear as swellings of the digit, often with loss of the nail and destruction of underlying bone, mimicking osteomyelitis. Radiographs and a biopsy are indicated for older dogs that present with a festering, swollen toe lesion. Click here to see a radiograph of oesteomyelitis in a dog's toe.

In contrast, the prognosis is good for dogs with digital squamous cell carcinoma (SCC). 95% of dogs with digital SCC survive 1 year after amputation and 75% survive 2 years after amputation. SCC have a low metastatic rate and most have not yet metastasized at the time of amputation and diagnosis. In addition to chest radiographs prior to surgery, local lymph node aspiration for cytology during surgery is important in order to help assess prognosis.
Term
A 30 pound (14 kg) 3 year old male neutered mixed breed dog is presented from Massachusetts in the northeastern United States. The dog is trembling and ataxic and has a seizure on the exam table during a physical exam.

The owner relates that he got into the garbage can 4 hours ago, which contained moldy bread, some bad salmon, a bag of chocolate-covered raisins, several super absorbent baby diapers, Styrofoam peanuts, a garbage-can deodorant cake and a dead toad.

Which item is most likely to be causing this dog's neurologic signs?


A - Dead toad
B - Deodorant cake
C - Rotten salmon
D - Super absorbent material in baby diapers
E - Chocolate-covered raisins
Definition
Answer: B- Deodorant cake
Paradichlorobenzene is an organochlorine insecticide, mainly affecting the CNS found in deodorizer cakes in diaper buckets, garbage cans and in bathrooms. Typically a problem of dogs, animals present with tremors, salivation, ataxia and seizures.

Toad poisoning mostly causes local (oral) irritant effects like profuse salivation, head shaking, pawing at the mouth, retching and sometimes vomiting. The marine toad, Bufo marinus, an introduced species established in Texas, Florida and Hawaii has highly toxic venom. Marine toads can cause life-threatening cardiac and CNS signs with 20-100% mortality in untreated cases.

Raisins and grapes can cause anuric renal disease in some dogs.

Don't confuse eating rotten salmon (GI upset) with salmon poisoning, a systemic infectious disease of dogs that usually occurs 5-7 days after eating infected fish It is associated with flukes and caused by Neorickettsia helminthoeca.
Term
A 6-year old Standardbred jumper is presented with a 3-month history of poor performance and intermittent shifting hindleg lameness. On physical exam, there is poor muscling of the gluteal muscles and some asymmetry to the croup (rump).

The horse has a 7 cm. swelling over the hindquarters on the left dorsal side and shows pain and a reluctance to ventroflex the back when midline pressure is applied. On rectal palpation crepitation and shifting can be felt dorsally as the horse walks slowly forward.

What is the diagnosis?

A - Fibrotic ossifying myopathy
B - Coxofemoral luxation
C - Overlapping vertebral spinous processes
D - Sacroiliac subluxation
E - Croup myopathy (longissimus dorsi, supraspinous ligament)
Definition
Answer: D- sacroiliac subluxation
Think of sacroiliac subluxation or sprain in an athletic jumping horse with:
--Intermittent hindlimb lameness localized in the croup (rump)
--Evidence of back pain
--Swelling over the tuber sacrale (hunter's bumps)
--Crepitance felt rectally in the sacroiliac area.

Horses with hunter's bumps can be sound if the injury has completely healed, but lameness can be observed if the injury is recent or has never resolved. Sometimes confused with stifle lameness.

Coxofemoral luxation is rare in horses due to a deep acetabulaum and the presence of the accessory ligament of the hip, unique to horses.

Overlapping vertebral spinous processes is a problem of thoracic and lumbar vertebra under the saddle area, seen in short-backed eventing horses (hunter/jumpers, dressage).

Back muscle and ligament strain is the #1 cause of back pain in horses, accounting for 33% of cases. Most commonly associated with the longissimus dorsi muscle, sublumbar illiopsoas muscles and the supraspinous ligament and also seen in eventing horses.
Term
Several sheep are sick at a petting-zoo that has cows, horses, pigs, bison and white-tailed deer. One of the deer is also affected. Affected sheep are listless and off-feed, with serous or mucopurulent nasal discharge and rectal temperatures ranging from 105-107.5°F (40.5-42°C).

Physical exam shows swollen muzzles with edema and congestion of the lips, nose and face with small hemorrhages and ulcers on the mucous membranes. The ulcers appear where the teeth contact the swollen lips and tongue. Two affected sheep are lame.

What is the diagnosis?

A - Bluetongue
B - Peste des petits ruminants (PPR)
C - Contagious ecthyma (soremouth)
D - Caprine arthritis encephalitis
E - Foot-and-mouth disease (FMD)
Definition
Answer: A - bluetongue
Swollen sore muzzles with mucous membrane erosions, high fevers and lameness suggests infection with Bluetongue virus. Bluetongue is almost exclusively seen in sheep, though white-tailed deer, pronghorn antelope and desert bighorn sheep in North America can be severely affected. Rare in cows. REPORTABLE.

Bluetongue is indistinguishable from Foot and Mouth disease (FMD), so that is a good second choice. But FMD is unlikely in the scenario above because FMD mainly affects pigs and cattle.
Term
Which choice is not one of the four core feline vaccines?

A - Panleukopenia
B - Feline calicivirus
C - Feline herpesvirus-1
D - Feline infectious peritonitis
E - Feline viral rhinotracheitis
Definition
Answer: D- feline infectious peritonitis
Feline infectious peritonitis (FIP) is not a core feline vaccine.

According to American Association of Feline Practitioners (AAFP) vaccination guidelines, there are 4 core feline vaccines-
Rabies
Feline calicivirus
Feline panleukopenia
Feline viral rhinotracheitis (herpesvirus-1).

Although Feline leukemia (FeLV) vaccine is not a core vaccine, it is highly recommended for kittens.
Term
A dog hit by a car presents with stiff hypermetric forelimbs and flaccid, paralyzed hindlimbs.

Where is the lesion likely to be?

A - Cervicothoracic: C6-T2
B - Thoracolumbar T3-L3
C - Lumbosacral L4-S3
D - Cannot tell without more information
E - Cranial cervical: C1-C5
Definition
ANSWER: B - thoracolumbar T3-L3
Thoracolumbar T3-L3 . This is likely to be Schiff-Sherrington syndrome ie: severe spinal cord trauma T3-L3, with thoracic limb extensor rigidity and hind limb flaccid paralysis. Lesion is caudal to T2, but inhibitory neurons in lumbar spinal cord (especially L2-L4) affect neurons in the cervical intumescence ( C6-T2).

With severe trauma T2-T13, inhibitory pathways are interrupted; Cervical intumescence neurons are "released" and cause extensor hypertonia in the forelimbs.

Can localize by checking cutaneous trunci reflex-The lesion is usually 1-2 vertebrae cranial to the line of analgesia (where dog does not feel pinching skin).
Term
You are called to examine an unthrifty-looking 5-month old steer that is failing to gain weight. He appears somewhat lame, has a rough, light-colored hair coat and depigmented hair around his eyes like spectacles. The steer has a watery dark diarrhea full of gassy bubbles. What is your diagnosis? A - Quercus (Oak) poisoning B - Molybdenum deficiency C - Magnesium deficiency D - Copper deficiency E - Selenium toxicity
Definition
Answer: D- Copper deficiency This is copper deficiency, which presents with "Ain't Doin' Right" ( ADR ) signs : ACHROMOTRICHIA (depigmented hair, especially around the eyes= "SPECTACLES") rough coat, decreased milk yield, lameness and decreased fertility, libido and "Peat Scours" also called "Teart" (severe scours with gas bubbles). Molybdenum toxicosis causes a secondary DEFICIENCY in Copper. Oak poisoning is more a gradual, multisystemic disease (nephrotoxic, hepatotoxic), can see PU/PD, hematuria, weight loss. Tetany is the classic presentation of Hypomagnesemia in cattle, along with hyperexcitability, ataxia, convulsions and death. Selenium toxicosis has many presentations depending on species, dose and length of time exposed.
Term
[image]
Definition

This is a classic history for acquired patent urachus. The dripping tissue tag in front is the umbilicus; penis is caudal. Follow this link to see an image of equine umbilical anatomy.

Recall that the urachus connects fetal bladder to allantoic sac by way of the umbilicus during gestation. Normally, urachus closes in the first days after birth.

A COMMON problem. Can be congenital, but MOSTLY associated with EXCESSIVE HANDLING, (ie: improper lifting, handling umbilicus too much). In fact, patent urachus develops in high % of foals in neonatal intensive care. Also associated with umbilical infection. One review of 16 foals with umbilical infections found 13 (81%) also had patent urachus.
With Uroperitoneum, look for stranguria, enlarging abdomen.
Umbilical hernias may become hot and infected, and possibly leak a little pus, but are not characterized by clear fluids leaking out for days.
Expect urine leaking from the penile anatomy, not the umbilicus with Hypospadias.

Term
[image]
Definition

This is Dorsal displacement of the soft palate (DDSP). The caudal free margin of the soft palate moves dorsal to epiglottis, obstructing the airway and causing exercise intolerance. Rx conservatively, eliminating possible contributing diseases first (ie: rest, anti-inflammatories). Surgical treatments (Sternothyrohyoideus myectomy or soft palate resection) have mixed success rates around 50%.

Epiglottic entrapment is a big DDX for DDSP. Outline of the epiglottis can still be seen with epiglottic entrapment, UNlike DDSP.

Cleft palate is a newborn disease. See difficulty suckling, dysphagia, MILK DRIPPING from NOSTRILS. Click here to see a Cleft palate. Euthanize if severe. Surgical closure if small.

Laryngeal hemiplegia ("Roarers") present with inspiratory noise during exercise and exercise intolerance. Click here to see laryngeal hemiplegia. More than 90% occur on LEFT side. Rx is surgery.

Pharyngeal lymphoid hyperplasia (PLH) is common. Thought to be a normal immunologic event in younger horses.

Term
[image]
Definition
This is the cerebrocortical necrosis of Polioencephalomalacia (PEM), basically a nutritional disease. PEM is traditionally associated with LOW THIAMINE but increasingly associated with HIGH SULFUR diets. 

PATHOGNOMONIC dorsomedial strabismus ("Stargazing"), history and cerebrocortical necrosis that lights up under UV light all suggest PEM.
Term
[image]
Definition

Answer: FOWL CHOLERA

This is Fowl Cholera, caused by Pasteurella multocida. Think sudden onset septicemia. Signs VARY greatly. In acute fowl cholera, dead birds are first indication of disease. May see clinical picture described above. with chronic cases, see localized infections: Sternal bursae, wattles, joints, tendon sheaths, footpads SWELL with fibrinosuppurative exudate. See torticollis if meninges, middle ear, cranial bones infected. Follow this link to see original Merck image of Fowl Cholera.

With Infectious Coryza think acute respiratory disease with nasal discharge, sneezing, andswelling under the eyes, caused by Haemophilus paragallinarum (gallinarum). Follow this link to see an image of Infectious Coryza.

With Infectious Laryngotracheitis (ILT) look for gasping, coughing, blood stained beaks, blood occluding trachea on necropsy. In most states ILT is REPORTABLE. Follow this link to see an image of ILT.

Infectious Bronchitis is characterized by respiratory signs, decreased egg production and poor egg quality. Classically may see "wrinkled eggs" with inf Bronchitis Newcastle diseaseis characterized primarily by respiratory signs. Severe forms include depression, neurologic signs or diarrhea. Look for GI hemorrhage with most severe form, Viscerotropic Velogenic Newcastle disease (VVND), which is REPORTABLE!

Term
You are presented with a 10-year old male intact Doberman mix dog with one testicle that is grossly enlarged and nonpainful and the other atrophied. What condition(s) do you expect to see associated with this presentation?

A - Penile hypertrophy, hirsutism
B - Increased aggression, territoriality
C - Benign prostatic hypertrophy, constipation
D - Hypothyroidism
E - Feminization syndrome
Definition

Feminization syndrome. Nonpainful unilateral testicular swelling in an old dog should make you think of neoplasia, especially an estrogen-secreting sertoli cell tumor, which can cause feminization syndrome. See gynecomastia, penile atrophy, pendulous prepuce, attraction of other males, blood dyscrasias, +/-bone marrow depression (via high estrogen), bilateral alopecia. 

Remember that cryptorchidism increases risk of sertoli cell tumor: About 25-29% of sertolis develop feminization syndrome & ~ 70% of intra-abdominal testicular tumors, regardless of type, cause feminization syndrome. 

May see prostate hypertrophy with sertoli, but it is due to squamous metaplasia, not the classic (and common) benign prostatic hypertrophy (BPH). Hypothyroidism and BPH are common older dog conditions, but not associated with unilateral testicular hypertrophy. 

Remember that sex hormones are also secreted by the zona reticularis of adrenal cortex and can see feminization or virilism with excess secretion. 

Term
[image]
Definition

Answer: BLACKLEG

This is Blackleg, caused by Clostridium chauvoei (feseri). Look for peracute, febrile disease. May have acute lameness and a few animals found dead. Some may have edematous, crepitant ("popping, crackling" sound/feel) swellings. Death within 12-48 hrs. Beef breeds mostly, best animals in herd, 6 mos-2 yrs, summer and fall, rarely in winter.

The blackleg organism is a normal inhabitant of gut, infection develops WITHOUT wounds, though bruising may precipitate disease. (In contrast to tetanus (Clostridium tetani), which enters via a wound, and causes disease more typically an individual sick cow, not an outbreak).

DDX includes: 
Anthrax (Bacillus anthracis, reportable) 
Lightning strike (sudden death, single animal)
Bacillary hemoglobinuria (Clostridium hemolyticum)
Malignant edema (Clostridium septicum). 

Term
A 5-year-old neutered domestic shorthair cat is presented for lethargy, vomiting and diarrhea. The owner saw the cat licking up antifreeze spilled while he was flushing his truck's radiator yesterday.

The cat is markedly depressed and dehydrated on physical exam.

Which of the following choices is the most appropriate treatment?

A - Ethanol 20%
B - Pralidoxime chloride (2-PAM)
C - Atipamezole, IV lactated ringer's
D - Calcium EDTA in 0.9% NaCl
E - Activated charcoal
Definition

Answer: A - Ethanol 2%

More than 3 hours after ingestion, ethanol is still the treatment of choice for cats with ethylene glycol toxicity.

Recent clinical trials suggest that Fomepizole (4 methyl pyrazole, 4-MP) can be a more effective treatment than ethanol in cats when administered
-- At high doses (extra-label) and 
-- Within 3 hours of ingestion of ethylene glycol.

Do 4 things when you see a case of ethylene glycol toxicity:
1. Decrease EG absorption: Induce vomiting +/- gastric lavage (or both) followed by activated charcoal, sodium sulfate within 1-2 hr of ingestion. 
2. Promote diuresis: IV fluids 
3. Address metabolic acidosis: Add bicarbonate in fluids 
4. Prevent metabolism of EG: 4-MP or ethanol treatment.

Term
A 5 year old female spayed Cocker Spaniel is presented with a strange expression. Her right ear and lip appear to droop. There is ptosis O.D. (right eye) and the dog is drooling on the exam table.

What anatomic structure is damaged?

A - Right side inner ear
B - Trigeminal nerve
C - Facial nerve
D - Left side inner ear
E - Left side medulla, motor tract
Definition

Answer: C - Facial Nerve

Think of facial nerve paralysis (CN 7) with a unilaterally droopy face. 

Remember the facial nerve is motor to the muscles of facial expression (explaining the right side drooped ear, lip and eyelid) and innervates the lacrimal and salivary glands. Loss of innervations can lead to a dry eye, and possibly to exposure keratitis if animal losses ability to close eyelid from damage to facial nerve innervation of the orbicularis oculi muscle. 

Idiopathic in 75% of canine cases (25% of cats). Can also see these signs with middle ear damage (from otitis media), from facial nerve trauma (ear surgery in dogs, or pressure from halter buckles in anesthetized horse), or neoplasia. Follow this link to see a horse with facial nerve damage: note the nose pulled to horse's left. (means facial nerve damaged on right). 

Think more of a dropped jaw with trigeminal nerve neuropathy (CN 5-dogs, horses).

Term
You suspect hyperadrenocorticism in a 9-year old female spayed dog with a two-month history of increased appetite, thirst and urinary accidents.

Which diagnostic test do you trust the most if it has a positive result?

Urine Cortisol Creatinine Ratio (UCCR). Sensitivity=95% , Specificity =25%
ACTH Stimulation. Sensitivity=80% , Specificity =85%
Low Dose Dexamethasone Suppression. Sensitivity=90% , Specificity =50%

A - Cannot say without knowing the predictive value negative
B - Low Dose Dexamethasone Suppression (LDDS)
C - Urine Cortisol Creatinine Ratio
D - ACTH Stimulation
E - Cannot say without knowing the predictive value positive
Definition

Answer: D - ACTH Stimulation.

The two diagnostic screening tests used most for diagnosis of hyperadrenocorticism are the ACTH stimulation test and the LDDS. Remember your Ps andNs. 

ACTH Stim is the most sPecific (fewer false Pos, so trust a POS test more). 
LDDS is more seNsitive (fewer false Negs, so trust NEG test more).

Either way, these 2 tests are good to screen for Cushing's, but a confirmatory test is usually called for if you get a positive, to differentiate between a pituitary HAC or an adrenal HAC case [high-dose dexamethasone suppression test (HDDST) or ACTH concentration]. 

The Urine Cortisol Creatinine Ratio (UCCR) is a poor screening test because it has low specificity (~25%), which means high false positives. 75% of dogs with non-adrenal illness will have a UCCR result consistant with HAC. 

Term
NAVLE Question of the Day:

You are presented with a dog with the following findings.

Blood Gas: PCO2-=28.3, pH=7.27, PO2=85.2, HCO3-=13.9

Blood chemistry: Na=136.2, Cl=91.3, Ca=9.1, K=3.5, P=9.3, TCO2=14.2, ALT=331, LDH=354, AST= 14, SDH =6, GGT=20, Glucose=150, Alk Phos=1018, BUN=2.8, Bilirubin (total) = 0.3

What is the anion gap?

A - 28.3
B - 34.5
C - 28.1
D - 31.2
E - 42.2
Definition

ANSWER: B 34.5 
Anion gap= (Positives)-(Negatives) = (Na+ and K+) -(Cl- and HCO3-).
If you did not do a blood gas analysis, then substitute TCO2 for HCO3-, and the AG value is about the same, (34.2 in this case).

Anion Gap (AG) = [136.2 (Na+) + (3.5(K+)] - [91.3 (Cl-) + 13.9(HCO3-)]
AG= [139.7 cations] - [105.2 anions]=34.5 anion gap

Typically see increased anion gap with:
Diabetic ketoacidosis
Renal insufficiency
Ethylene glycol toxicity
Lactic acidosis from grain overload or strenuous exercise

Decreased anion gap is UNcommon. Think hemodilution, hypoalbuminemia, hypercalcemia.

Term
An 8-year old neutered male golden retriever is presented with a 2 month history of nonspecific lethargy and weight loss.
T= 99.8 F (37.7 C) RR= 24 HR = 64 bpm
On physical exam, there is normal hydration and mild ascites.
CBC: PCV=45 % RBC 6.5 x 1012 g/L normocytic, normochromic
WBC=12,350 Neuts=63% Lymphs=24% Monos=5% Eos=8%

Blood chemistry reveals the following:

Na= 148 mEq/L, Cl=118 mEq/L, K= 6.0 mEq/L
LDH=200 u/L, ALT=36 u/L, AST=14 u/L, GGT=4.4 u/L, Glucose=120 mg/dL, Alk Phos=101 u/L
Cholesterol=200 mg/dL, BUN= 29 mg/dL, Creatinine=1.6 mg/dL, Bilirubin (total)=0.4 mg/dL
Total protein=4.6 g/dL , (globulin=3.0 g/dL, albumin=1.6 g/dL)
Urinalysis: U Sp. G= 1.031, no WBC or RBC, protein +++
Which diagnosis is on top of the differential diagnosis list?
A - Diabetes insipidus
B - Renal failure
C - Glomerulonephritis
D - Pyelonephritis
E - Hypoadrenocorticism
Definition

Answer: C - glomerulonephritis 

Glomerulonephritis is a top rule out here for a middle-aged dog with low albumin, ascites and proteinuria without evidence of urinary infection (no RBC or WBC in urine sediment). 

This is a type III immune reaction in which antigen-antibody complexes deposited on glomeruli or capillary walls of glomeruli, stimulate complement and cause damage via a neutrophilic inflammatory response. 

Another big rule out for a canine protein-losing nephropathy is renal amyloidosis. Renal biopsy is the best way to distinguish between glomerulonephritis and amyloidosis 

Term
[image]
Definition

Answer: D Use High-dose antibiotics for sick piglets.


This is Exudative dermatitis ("greasy pig disease"), caused by Staph hyicus. Responds to most antibiotics (pen, amoxi, TMS, erythromycin, lincomycin, tylosin, AGs, cephs) at high dose 7-10 days, plus topical antiseptic.

In severe outbreaks, ABX for contact pigs recommended for a few days. Disinfection, good environmental hygiene important.

In chronically affected herds, autogenous bacterins have been used with some success to decrease incidence.

Term
Foal heat diarrhea is typically associated with which choice?

A - Warm and humid weather
B - Hemorrhagic enteritis
C - Neutropenia and fever
D - Decreased suckling
E - Alterations in diet
Definition

Answer: E - Alternations in diet 

Mild, self-limiting diarrhea in a foal 7-14 days of age is called foal heat diarrhea, because it coincides with the first estrus cycle post-foaling in the dam. Causes of foal heat diarrhea are poorly understood, but are thought to be related to a foal's tendency to start sampling hay and grain and practice coprophagy by 5 to 7 days of age, with consequent alterations in bacterial flora. 

Foals are active and alert, with a normal appetite and vital signs. Clinical signs such as fever and lethargy, hematochezia or melena, and laboratory findings such as neutropenia are notroutinely observed.

Term
[image]
Definition

Answer: Inguinal hernia

This is an Inguinal hernia in a piglet. A COMMON problem in pigs, Rx surgically: midline skin incision, cranial to scrotum; ligate and excise vas deferens, blood vessels. BOTH inguinal ring areas should be closed to prevent herniation post-surgery. Removal of tunic, cremaster muscle, extra subQ tissue, with closure to obliterate empty space helps prevent seromas.

Follow this link to a good Merck diagram of inguinal hernia anatomy in a horse.

Intersex syndrome is described in pigs and goat (rare), but look for BOTH genitalia.

Perineal hernia more a problem of middle-aged pure bred dogs.

With Cryptorchidism a testicle is RETAINED, not protruding.

Term
Which of the following represent the most effective control measures in herds infected with porcine reproductive and respiratory syndrome (PRRS) virus?

A - Buy PRRS-negative gilts, test on arrival, segregate 45 d, retest before join herd
B - Nursery depopulation, segregated early weaning 12-18 mos after outbreak
C - Maintain therapeutic levels of streptomycin in feed
D - Cull all sows after first farrowing
E - Vaccinate boars in contact with sows, maximize pen ventilation
Definition

Answer: B 

12-18 months after the initial outbreak, sows are usually no longer transmitting the Porcine reproductive and respiratory syndrome (PRRS) virus, so segregated early weaning and removal of nursery pigs (with concurrent disinfection of nursery before repopulation) may help control the virus. 

Because it appears that sows generally only abort once, culling after the first farrowing not advisable.

Antibiotic administration can help control secondary bacterial infections, but not the arterivirus responsible for PRRS. Adding negative gilts to a herd already infected with the virus is unlikely to result in control.

Term
[image]
Definition

Answer: A - Antihelminthics are the treatment of choice.

This is ascarid (roundworm) infestation and these are classic "milk spots"- liver scars left by migrating ascarid larva traveling to the lungs. In heavy infestation, larvae can cause pulmonary edema, consolidation, severe respiratory distress. May see icterus. Follow this link to seeanother image of milk spots. A fecal exam would show eggs that look like this.

Rx with antihelminthics, like benzimidazoles, ivermectin, pyrantel, levamisole. May need antibiotics to treat secondary bacterial pneumonia, but primary Rx is antihelminthics.

Lincomycin is a lincosamide antibiotic used in pigs against mycoplasma pneumonia.

Morantel tetrahydropyrimidine is an antihelminthic feed additive for ruminants, not labeled for pigs.

Term
A 4-year old quarterhorse mare is presented with a runny left eye and a urine-scalded perineum. No other horses on the farm are sick. Physical exam reveals a corneal ulcer and keratitis OS (left eye), and atrophy of the temporal and masseter muscles.

There is decreased perineal sensation, a weak tail and weak anal sphincter with retained manure. The horse is bright, alert and responsive. T= 102.2 F (39.1 C) HR= 40 RR= 12

Which of the following is the most likely diagnosis?

A - Nigropallidal encephalomalacia
B - Botulism
C - Cauda equina neuritis
D - Equine degenerative myelopathy (EDM)
E - Equine protozoal myelopathy (EPM)
Definition

Answer: C - Cauda equina neuritis

Horses with Cauda equina neuritis (also called polyneuritis equi) have a progressive symetric LMN paresis of the tail, bladder, rectum, anal sphincter. Look for urinary incontinence, fecal retention and a weak or paralyzed tail. May see hind limb paresis if lumbosacral spinal cord is affected. 

Cranial nerves can also be affected, but typically cranial involvement is asymetric. May see temporal or masseter atrophy (Cranial Nerve 5), facial paralysis and exposure keratitis (Cranial nerve 7), head tilt or other CNS signs.
Cause is unknown, may be an autoimmune process. Grave prognosis. Eventually euthanized.

Herpesvirus myeloencephalopathy (EHV-1) may also present with urinary incontinence, but this is an uncommon manifestation of equine rhinopneumonitis. You would expect to hear a history of the more common EHV signs in other horses from the same farm, like respiratory disease ("snots") in foals and abortions in mares.

Term
[image]
Definition

Answer: E - Penicillin IM in acute, cull chronics

Penicillin is the treatment of choice for acute cases. CULL Chronic cases. These are ACUTE cases of erysipelas

Lincomycin is more frequently used to treat respiratory disease and the polyarthritis associated with Mycoplasma hyosynoviae.

Term
An adopted adult cat from a shelter with an unknown vaccination history tests positive for feline leukemia virus (FeLV) infection by IFA.

What do you tell the owner?

A - Possible transient FeLV infection
B - Cat may be vaccinated for FeLV
C - Need an ELISA test in 12 weeks to confirm diagnosis
D - Strong chance of lifelong FeLV infection
E - Need a Western blot test to confirm diagnosis
Definition

Answer: D - strong chance of lifelong infection

97% of cats positive for feline leukemia virus (FeLV) by IFA remain persistently infected and viremic for life.

The FeLV ELISA is more seNsitive than the IFA (fewer false Negs, so trust a NEG test more), so it is a better general screening test with which to start. Click here for a diagram.

The FeLV IFA is more sPecific than the ELISA, (fewer false Pos, so trust a POS test more) so it is a better confirmatory test for any cats with a positive FeLV ELISA. Click here for a diagram.

THREE things to remember about FeLV and vaccinations:
1. FeLV ELISA and IFA tests measure ANTIGEN, not antibody, so FeLV vaccination does NOT interfere with testing. 
2.Vaccinate FeLV-positive cats yearly against respiratory, enteric viruses with inactivatedvaccines.
3. FeLV vaccination for FeLV-negative cats has been associated with sarcomas. VaccinateLOW on the LEFT hind.

Term
[image]
Definition

Answer: C - hypertrophic cardiomyopathy

This is the classic "Valentine heart" of feline Hypertrophic Cardiomyopathy (HCM).Note the very high RR and HR. Typically see HCM in cats 5-7 years old, more often in males.

Cause unknown. Look for thickened L. ventricle wall on echocardiography. Follow this link tosee thickened L ventricle on necropsy

May see secondary pulmonary hypertension, edema, pleural effusion. DDX includes hyperthyroidism, systemic hypertension, acromegaly, congenital aortic stenosis.

Systolic murmur loudest on L betw 5th-6th suggests MITRAL valve insufficiency (left AV). Aortic insufficiency is DIASTOLIC. Tricuspid valve murmurs (right AV) are loudest on the RIGHT (See Blackwell's, p. 905, Tschauner p. 212 for excellent murmur DDX tables).

Taurine deficiency is associated with DILATED cardiomyopathy (DCM) in cats. Think more of LARGE BREED DOGS with DCM.

Technically, HCM is a form of congestive heart failure, but the "Valentine" radiograph points you to the much more specific diagnosis of HCM.

Term
[image]
Definition

Answer: A - photosensitization

This is photosensitization of non-pigmented skin. Note how the brown areas are more protected, but the skin of the white muzzle has sloughed off. Click here to seephotosensitization in a cow.

The most common causes are from toxic plants. Primary photosensitization is caused by photodynamic substances in the plant itself. Secondary photosensitization is caused by plant-related hepatic damage, which releases photodynamic substances like phylloerythrin, a breakdown of product chlorophyll. Secondary photosensitization is much more common than primary.

Examples of primary photosensitizers are hypericin from Hypericum perforatum (St. John's wort) and fagopyrin from Fagopyrum esculentum (buckwheat).

Causes of secondary photosensitization include common bile duct occlusion, facial eczema(pithomycotoxicosis) and mycotoxic lupinosis and pyrrolizidine alkaloids.

Term
[image]
Definition

Answer: B - Anterior lens luxation

This is Anterior lens luxation. Follow this link to see another lens displacement image- note the aphakic crescent to right side (area of pupil without lens) 

Look for an opaque (cloudy, milky) lens behind pupil with cataracts. In contrast, Nuclear sclerosis looks like this

Orbital cellulitis presents in large and hunting breeds of dogs with acute pain on opening mouth, eyelid swelling, unilateral prolapse of nictitating membrane, eye protrusion, conjunctivitis. Dog's face looks swollen with inflamed, red eyes , (aka: "Devil dog") 

Term
Which of the following choices is the most appropriate next step after a 3-year old stray cat tests positive for Feline Immunodeficiency Virus (FIV) on a routine FIV ELISA screening test?

A - Repeat ELISA to rule out false positive
B - Recheck serum by FIV complement fixation test
C - Euthanize
D - Pull a blood sample for a Western Blot FIV test
E - Pull a blood sample for FeLV Antigen test
Definition

Answer: D - Western Blot FIV test 

Pull a blood sample for a Western Blot FIV test. The feline immunodeficiency virus (FIV) ELISA is the standard first-step screening test to detect if the cat has been exposed to the FIV virus and has circulating antibody. A confirmatory Western blot antibody test for FIV is the standard confirmatory test. 

A Western blot test is especially important in areas with low FIV prevalence, where the risk of of false positive FIV ELISA is higher. Click here to see table that shows why a test's predictive value goes down as prevalence goes down

Remember that cats vaccinated for FIV will test positive for FIV antibodies by FIV ELISA and Western blot. 

This is DIFFERENT from feline leukemia virus (FeLV) testing because the FeLV ELISA and IFA tests measure ANTIGEN, not antibody, so FeLV vaccination does NOT interfere with testing. 

Term
[image]
Definition

Answer: A - Thyroid testing

Thyroid function, BUN/Creatinine. This presentation of vision loss in an older cat suggestsretinal detachment, typically secondary to HYPERTENSION. 

Chronic renal disease may lead to HYPERtension and retinal detachment-Physical exam, history and BUN/creatinine will rule this disease in or out.

HYPERthyroidism can ALSO cause HYPERtension. Your physical exam and history, plus a measure of T4 will help you evaluate this condition.

Other diseases associated with retinal detachment include your alphabet diseases and 2 T's:
FIP, FeLV, FIV
Toxoplasmosis and Trauma 

Follow these links to see classic retinal detachment in a dog with renal disease and focal retinal detachment in a cat with cryptococcosis.

Remember you can see hemorrhage with retinal detachment, as in this example from a young dog with Collie-eye anomaly.

For comparison, see this example of another big retinal disease, Progressive retinal atrophy(PRA)- think of SLOW loss of vision, starting with night vision loss and progressing to blindness over MONTHS. 

Term
A 9 year old German shepherd is presented with unchecked bleeding from a cut on the gums above the right canine tooth. The owner relates that the dog has lost weight and had an episode of collapse a 3 days ago, but he recovered.

On physical exam, the gums are pale with petechiae and ecchymotic hemorrhages. There is tachycardia and a palpable cranial abdominal mass.

A coagulation profile shows the following:

Thrombocytes= 82,533 per microliter
Buccal mucosal bleeding time (BMBT), increased
Activated partial thromboplastin time (APTT), increased
Prothrombin time (PT), increased
Thrombin time (TT), increased
Fibrin degradation products (FDPs), increased

What disorder of coagulation best fits this pattern?

A - Von Willebrand's disease
B - Anticoagulant rodenticide toxicity
C - Idiopathic thrombocytopenia
D - Hepatic insufficiency
E - Disseminated intravascular coagulation (DIC)
Definition

Answer: E - DIC 

A lab pattern of low platelets, increased bleeding time and across the board increases in aPTT, PT, TT and FDP tests suggests disseminated intravascular coagulation (DIC). DIC is not a disease in its own right- it is a complex hemostatic defect characterized by enhanced coagulation and fibrinolysis, secondary to other diseases. Fibrinolysis and depletion of clotting factors leads to hemorrhage. 

Many, many diseases, all of them bad, can precipitate DIC. This case presentation (pale, older German shepherd with Hx of collapse, bleeding and an abdominal mass) suggests hemangiosarcoma. 

Remember your "H diseases" associated with DIC:
Heartworm
Heart failure
Hemolytic anemia
Hemangiosarcoma
Hemorrhagic gastroenteritis
Hepatic disease, especially hepatic lipidosis in cats. 
Gastric dilatation-volvulus (GDV), mammary gland carcinoma and pancreatitis can also lead to DIC. 

Term
Which of the following antimicrobials are best indicated for use against anaerobes?

A - Amikacin, Neomycin, Cephalosporins
B - Enrofloxacin, Aminoglycosides, Chloramphenicol
C - Cephalexin, Chloramphenicol, Clindamycin
D - Amikacin, Enrofloxacin, Metronidazole
E - Clindamycin, Metronidazole, Gentamycin
Definition

Answer: C - Cephalexin, Chloramphenicol, Clindamycin

Think of the "3 C's" against anaerobes (plus Flagyl®).

Chloramphenicol (CAPL), Cephalexin (and all other cephalosporins), Clindamycin(Antirobe®) and Metronidazole (Flagyl®) are all used against anaerobes. 

A common use of Antirobe®, for example, is for treatment of abscesses or bites wounds in cats.

Enrofloxacin (Baytril®), and Aminoglycosides (ie: Gentamycin, Amikacin, Neomycin) are two to think of which DO NOT fight anaerobes. (better for gram negatives).

Term
[image]
Definition

 

Answer: A - Type III

In type III reactions antigen-antibody complexes are deposited on the endothelium, stimulating complement and a neutrophilic inflammatory response and vascular damage. Look for localized or multisystemic vasculitis. 

Classic type III diseases include 
Glomerulonephritis,
Hypersensitivity pneumonitis (think moldy hay),
Purpura hemorrhagica (think post-strangles) and
Anterior uveitis.

Term
Which of the following choices describes the correct order of putting on surgical attire and scrubbing for surgical personnel?

A - Hand scrub, gown, gloves, hair cap, facemask
B - Facemask, hair cap, gown, gloves, hand scrub
C - Gown, hand scrub, gloves, hair cap, facemask
D - Facemask, gown, hand scrub, gloves, hair cap
E - Hair cap, facemask, hand scrub, gown, gloves
Definition

Answer: E

The basic order is: Hair cap, facemask, hand scrub, gown, gloves. Most facilities have disposable shoe covers to wear, which are put on when you put on the hair cap and facemask. 

Gloving is important, but is not a substitute for proper scrubbing. There are three gloving methods:
1. Closed gloving (your scrubbed hand is inside the gown as you handle the glove).
2. Open gloving (your scrubbed hand touches the inside of the glove as you put it on).
3. Assisted gloving (an assistant who is already gloved holds your glove open as you put your scrubbed hand in).

Term
What is the earliest stage post-breeding that an experienced practitioner can reliably feel placentomes rectally in the gravid uterine horn in a pregnant cow?

A - 70-75 days
B - 30-35 days
C - 80-85 days
D - 40-55 days
E - 90-120 days
Definition

Answer: A - 70-75 days 

An experienced practitioner can reliably feel placentomes around 70-75 days. You can reliably feel the "membrane slip" of chorioallantoic membranes rectally at 30-35 days and fremitus (vibration in uterine artery of the gravid uterine horn) between 90-120 days.;

One way to remember these is that the EARLIEST occurs in REVERSE alphabetical order ie: 
Slip 30-35 days 
Placentomes 70-75 days 
Fremitus 90-120 days 

Term
[image]
Definition

Answer: D - Check the pasture for oak trees and acorns.

This is a classic case of acorn/oak bud toxicosis(Quercus spp). The sick animals have tubular nephritis evidenced by high urinary protein, casts, low urine specific gravity, and high BUN and creatinine. The nephritis is caused by hungry heifers eating green oak leaves that would have sprouted in the Spring.

The deformed calf is called an "acorn" calf. These calves are the result of a cows on a low protein diet coupled with acorn ingestion in mid to late pregnancy (in the Fall).

Pigweed (Amaranthus spp) and lamb's quarter (chenopodium) are sources of nitrate toxicity. Look for a presentation involving hypoxia (muscular tremors, weakness, brown cyanotic mucous membranes) due to methemoglobinemia.

Lysine deficiency may contribute to necrotic ear syndrome in pigs. 

Selenium or vitamin E deficiency is associated with white muscle disease. Look for stiffness and sudden death due to myocardial lesions in newborn calves. 

Think of cerebellar hypoplasia or weak calves born with congenital BVD.

Term
Which choice is permitted for use in food animals?

A - Chloramphenicol
B - Dexamethasone
C - Diethylstilbesterol
D - Furazolidone
E - Estradiol cypionate
Definition

Answer: B - Dexamethasone

Dexamethasone is allowed in food animals, but may cause abortion in pregnant animals. Typically, use dexamethasone in cattle to induce parturition (20-30 mg, IM, given within 2 wk of normal term).

Click here to see a list of drugs prohibited in food animals (No extra-label use).

Diethylstilbesterol (DES) is banned for use in food producing animals and should never be used. 

Chloramphenicol has been associated with bone marrow suppression/aplastic anemia in exposed humans, and is contraindicated in food-animals.

According a 2006 report from the Food and Drug Administration (FDA) the use of Estradiol cypionate (ECP) in animals is illegal. ECP has been used as an estrogenic hormone for reproductive therapy in food animals, but even extra-label, this is not allowed.

Furazolidone a nitrofuran, is not allowed.

Term
[image]
Definition

Answer: A - anesthesia distal to tourniquet 

Anesthesia distal to tourniquet. This is an example of intravenous regional anesthesia, the anesthetic method of choice for most bovine digital surgical procedures. It has similar results to a nerve block, but is easier to do with reliable results. You need only one injection and no special knowledge of anatomy. 2% Lidocaine infuses down the venous system and anesthetizes the entire lower limb distal to the tourniquet within about 10 minutes. 

The amount of lidocaine needed is typically 10-30 ml and should not exceed a total dose of 9 mg/kg. Click here to see tourniquet sites and veins for intravenous regional anesthesia of the distal bovine hindlimb. 

Do not keep a tourniquet in place more than one hour. When surgery is complete, slowly release the tourniquet and then retighten it. If antibiotics are indicated, infuse them now then release the tourniquet after a few more minutes. 

Some references call this a "Bier" block, after the pioneering German physician August Bier, who reported on this form of anesthesia in 1908.

Lidocaine (and procainamide) are used to treat arrhythmias, but more for ventricular fibrillation in small animals and not administered from the foot. Think of quinidine sulfate to treat atrial fibrillation in symptomatic horses.

Term
How many cows should be in estrus each day in a pen of 100 non-pregnant heifer cows?

A - 1 to 3
B - 6 to 8
C - 4 to 6
D - 12 to 13
E - 9 to 10
Definition

Answer: 4-6 heifers should be in estrus on any given day 

The bovine estrus cycle lasts 18 to 21 days. Therefore in 100 cows, estrus would occur in a number of cows that is calculated as (100 ÷ 18=5.55) to (100 ÷ 21= 4.76). So the best answer is 4-6 cows per day.

Term
A 6-year old quarterhorse gelding is presented in September in North America with a three-day history of depression, poor appetite, fever and worsening gait. The owner says the horse seems weak on his hind legs, stumbles and sometimes presses his head against the wall of his stall.

The horse is ataxic and hypermetric in all four legs. Serum antibody titers to EEE, WEE, and VEE are low. A Western Blot test of CSF for antibodies to Sarcocystis neurona is negative.

Which of the following diagnostic tests should be performed next?

A - Serum AGID for equine infectious anemia
B - Serum IgM capture ELISA for West Nile Virus
C - CSF tap and test for Japanese encephalitis
D - MRI to rule out nigropallidal encephalomalacia
E - Plasma antibody test for St. Louis encephalitis
Definition

Answer: B - Serum IgM capture ELISA for West Nile Virus.

Think of the equine encephalidities in a febrile horse, especially in the fall months. Because serum antibody tests for EEE, WEE and VEE are low, West Nile virus encephalitis is the big remaining rule out. 

Look for variable and nonspecific signs like depression, low-grade fever and anorexia in combination with neurologic signs like head-pressing (image), ataxia (often hind-end weakness or paralysis) and visual impairment. IgM capture ELISA is the test of choice. 

Remember that in most states and provinces, you must REPORT a horse with clinical signs of encephalomyelitis, even if the test results are not in yet. 

With equine infectious anemia (EIA), think of recurrent fever, weight loss, dependent edema,petechial hemorrhages (image) and icterus.

Think of ataxia and atrophy with equine protozoal myeloencephalitis (EPM) due to Sarcocystis neurona.

Term
[image]
Definition

Answer: A - Good place to take a pulse 

 A convenient way to take the pulse of a horse is to place two or three fingers over the facial artery, in the notch of the mandible just in front of the masseter (cheek) muscle.


Click here to see a to see a video of a vet taking a horse's pulse.

There are 4 other places on a horse you can take a horse's pulse, but only one is also on the head:
-Transverse facial artery (caudal to lateral canthus of the eye)
-Median artery (proximal, medial forearm)
-Great metatarsal artery (hindlimb, between McIII and McIV)
-Digital artery (caudal pastern area, above hoof). 

Palpate the digital artery to feel the "bounding pulses" of equine laminitis

For a good diagram of all five spots, see Pasquini and Spurgeon's Clinical Anatomy, 11th ed. p. 404.

Term
Failure to control mild endemic respiratory disease in swine caused by Mycoplasma hyopneumoniae predisposes pigs to complications.

Exudative bronchopneumonia and polyarthritis are most commonly seen in herds with infected with both Mycoplasma hyopneumoniae and which other organism?

A - Bordetella bronchiseptica
B - Hemophilus parasuis
C - Swine influenza virus
D - Pasteurella multocida
E - Fusobacterium necrophorum
Definition

Answer: D - Pasteurella multocida

Pasteurella multocida infection in conjunction with Mycoplasma hyopneumoniae causes exudative bronchopneumonia, polyarthritis, and chronic lung lesions. Mycoplasma hyopneumoniae, (also called "Enzootic pneumonia") is a common, smoldering low-level illness. Stressors (parasites, other infections, even weather) can result in severe pneumonia. 

Best managed by decreasing stressors with improved ventilation and decreased overcrowding. In endemic herds, ABX for sick individuals (ie: lincomycin, tylosin, tiamulin, or a tetracycline) helps control illness, most likely by preventing secondary bacterial infection (likePasteurella). 

Bacterin vaccines give good protection, decrease signs (coughing). Pre-farrowing vaccination of sows decreases colonization in suckling piglets. 

Term
Which choice is not one of the four core vaccines for dogs?

A - Parvovirus
B - Bordetella
C - Distemper
D - Adenovirus
E - Hepatitis
Definition

Answer: B - Bordetella 

Although commonly used, vaccines against infectious tracheobronchitis (kennel cough), caused by Bordetella bronchiseptica and canine parainfluenza are not part of the core canine vaccination regimen.

According to American Animal Hospital Association (AAHA) vaccination guidelines,there are 4 core canine vaccines (scroll down to table 1):
Rabies
Infectious Canine Hepatitis (Canine Adenovirus Type 1)
Distemper
Canine parvovirus.

Many combination vaccines for dogs also include parainfluenza and leptospirosis components, but AAHA does not consider these core vaccines per se.

According to American Association of Feline Practitioners (AAFP) vaccination guidelines, there are 4 core feline vaccines:
Rabies, Panleukopenia, Herpesvirus-1 and Calicivirus. 

Term
Where should a feline leukemia (FeLV) vaccine be injected?

A - Above stifle, lateral left hind
B - Above stifle, lateral right hind
C - Cervical interscapular region
D - Below stifle, lateral left hind
E - Below stifle, lateral right hind
Definition

Answer: D - below stifle, lateral left hind 

Below stifle, lateral left hind. Feline leukemia virus (FeLV) and rabies vaccinations have been associated with sarcomas. You should always keep a record of where vaccinations were given. 

Remember your L's and R's for feline vaccination:
For FeLV, vaccinate LOW and LATERAL on the LEFT hind. 
For Rabies vaccinate low and lateral on the RIGHT hind. 

Here is a direct quote from the AAFP Feline Vaccine Advisory Panel Report
"FeLV or FIV antigen (plus any other antigen except rabies) should be administered subcutaneously (SC) on the lateral side of the left hind limb below the stifle joint (vaccine-associated sarcomas arising in the proximal femoral area are difficult to completely excise; placement of vaccines in this area is strongly discouraged)."

Term
Which reportable condition can affect cattle, but mainly causes disease in sheep?

A - Anthrax
B - Rinderpest
C - Malignant Catarrhal fever
D - Vesicular stomatitis
E - Bluetongue
Definition

Answer: E - Bluetongue

Bluetongue is almost exclusively a sheep disease, (but cattle and deer can get it). 

Rinderpest mainly affects cattle and is reported to be eliminated as of October 2010 by the the United Nation's Global Rinderpest Eradication Program. Because it is a classic severe and reportable vesicular disease, vets will likely need to keep rinderpest on their mental DDX list for years to come.

Pseudorabies is basically a pig pathogen. Can affect cows, but horses (and humans) are resistant 

Vesicular Stomatitis (VS) can occur in horses, pigs, cows. Remember the big 8 vesicular diseases: BVD, IBR, BPS, MCF, Bluetongue, VS, FMD, Rinderpest) 

Term
[image]
Definition

Answer: D - transitional cell carcinoma

This is likely to be neoplasia, specifically, a transitional cell carcinoma seen here in the trigone of the bladder after pneumocystogram (air in bladder) and here after injection of contrast media

Hematuria, pollakiuria, abdominal pain and bacterial cystitis in an older dog may also suggest urolithiasis, but if stones were visible on radiograph, they would be less likely to sit in the trigone. The most common form is struvite urolithiasis (generally radio-opaque), seen in 60% dogs, 90% cats. 
Click here to see a radiograph of a cat with urolithiasis.

With renal calculi, look for classic "staghorns" in the kidneys on DV rads.

With prostatic hypertrophy look for prostate displacing the bladder cranially into the abdomen. The oval closest to the pelvis is the prostate, the larger oval cranial to it is the bladder.

Term
A cat with a previous diagnosis of diabetes mellitus confirmed by persistent fasting hyperglycemia and persistent glycosuria is presented for a routine check-up. The owner relates that she ran out of injectable insulin two months ago, but the cat seemed to do fine without it, so she stopped giving insulin shots.

On physical exam, the cat appears healthy and a dipstick test shows a blood glucose level of 125 mg/dL (normal 61-132 mg/dL)

What is the most likely explanation?

A - Concurrent diabetes insipidus
B - Transient diabetes mellitus
C - Type I diabetes mellitus
D - Incorrect initial diagnosis
E - Insulin resistance
Definition

Answer: B - Transient Diabetes Mellitus 

20% of cats may have transient or subclinical diabetes mellitus characterized by resolution of the clinical weeks to months after beginning insulin treatment. Clinical disease may or may not recur in the future.

Non-insulin-dependent diabetes mellitus (type II) occurs in about 30% of cats at the time of diabetes diagnosis. There is a a reduced population of pancreatic Beta cells.

Cats with non-insulin-dependent diabetes mellitus may respond to a treatment regimen of weight loss, diet, oral hypoglycemic drugs, and correction of concurrent insulin antagonistic disease; that is, they can respond without injectable insulin.

Persistent fasting hyperglycemia (blood glucose greater than 200 mg/dL) and persistent glycosuria are confirmatory for diabetes mellitus, so "incorrect diagnosis" is a wrong answer choice in this case.

Term
What acid-base abnormality would be most likely in a dairy cow with a displaced abomasum?

A - Metabolic alkalosis
B - Respiratory alkalosis
C - Depends on severity of displacement
D - Respiratory acidosis
E - Metabolic acidosis
Definition

Answer: A - Metabolic Alkalosis 

Think of hypochloremic metabolic alkalosis due to hydrochloric acid (HCl) sequestration in a cow with a displaced abomasum

Abomasal hypomotility, ongoing HCl secretion into the abomasum and partial abomasal outflow obstruction all contribute. Metabolic alkalosis can also be due to HCl loss in monogastic animals who vomit. 

In a similar way, metabolic acidosis can be due to HCO3-loss from diarrhea or saliva loss if animal cannot swallow.

Term
A three year old male neutered Weimaraner dog is presented with a two day history of worsening anorexia, lethargy, cough and exercise intolerance.

On physical exam, the mucous membranes are pale with a few petechiae.
T= 99.8 F (37.7 C) RR= 24 HR = 144 bpm

A coagulation profile shows the following:

Thrombocytes= 343,500 per microliter
Buccal mucosal bleeding time (BMBT), normal
Activated partial thromboplastin time (aPTT), increased
Prothrombin time (PT), increased
Thrombin time (TT), normal
Fibrin degradation products (FDPs), normal

Which of the following is the most appropriate diagnosis?

A - Disseminated intravascular coagulation (DIC)
B - Idiopathic thrombocytopenia
C - Anticoagulant rodenticide toxicity
D - Von Willebrand's disease
E - Neonatal isoerythrolysis
Definition

Answer: C - Anticoagulant Rodenticide toxicity

Anticoagulant rodenticide toxicity (or liver disease) can demonstrate increased activated partial thromboplastin time (aPTT) and prothrombin time (PT). 

Sometimes a mild to moderate thrombocytopenia may occur (50,000-150,000/μl). Proteins induced by vitamin K1 absence or antagonism (PIVKA) will be prolonged. The PIVKA test is more sensitive than PT/aPTT but is not as widely available. A threefold increase in PT or PIVKA is supportive of anticoagulant rodenticide toxicosis.

Often there is no history of exposure to a rodenticide. Clinically, vague initial clinical signs like anorexia, lethargy and exercise intolerance may be the first manifestation of illness. With progression, you may see an acute hemorrhagic presentation.

Term
What are the Coagulation factor values in these 4 major clotting disorders:
DIC
Thrombocytopenia
Von Willdebrand's
Anticoagulant rodenticide/ Liver disease
Definition
[image]
Term
During a routine immunization visit for a 2 year-old neutered male Newfoundland dog, you detect a systolic murmur heard loudest on the left side of the chest between the 2nd and 5th intercostal (IC) space and at the thoracic inlet lateral to the trachea.

Which condition is highest on your differential diagnosis list?

A - Aortic stenosis
B - Mitral dysplasia
C - Patent ductus arteriosus (PDA)
D - Tricuspid dysplasia
E - Pulmonic stenosis
Definition

Answer: A - Aortic stenosis 

Aortic stenosis (also called sub-aortic stenosis [SAS]), is a systolic murmur which may be heard most loudly on the left chest between the 2nd and 5th intercostal (IC) space or at the thoracic inlet (lateral to trachea). 

Inherited in Newfoundlands. Predilection in many BIG BREEDS- German Shepherd, Golden Retriever, Boxer, Rottweiler. 

Mitral dysplasia and other mitral valve problems are heard further back on left at 5th-6th IC. More common in CATS. 

With Pulmonic stenosis see RIGHT ventricular hypertrophy, because pulmonic valves blocks outflow from R ventricle (mostly dogs). Follow this link to see a Merck image of Rt. ventricular hypertrophy. 

Tricuspid dysplasia is heard further back on RIGHT at 5th-6th IC. Uncommon. 

Expect a continuous murmur with Patent ductus arteriosus (PDA). Vast majority detected at first vaccination visit. 

Term
What is the most common cause of Horner's Syndrome in dogs?

A - Retrobulbar neoplasia
B - Brachial plexus avulsion
C - Neck bite wounds
D - Otitis media
E - Idiopathic
Definition

Answer: E - Idiopathic 

Horner's Syndrome is IDIOPATHIC in 50-93% of dogs (45% in cats).

See four things with Horner's, ALL associated with an eye:
1. MIOSIS (constricted pupil) 
2. PROTRUSION 3rd eyelid (nictitans) 
3. ENOPHTHALMOS (sunken eye) 
4. PTOSIS (drooped eyelid), +/- anisocoria. 

Remember "My 3rd Sunken Toe" (Miosis, 3rd lid protrudes, Sunken eye, Ptosis). 

Diagnose Horner's syndrome by history, with 3 of 4 signs. Look for CLASSIC INCREASED anisocoria in the dark. Other diagnostic tests depend on where suspect primary disease. 

If you can get a primary diagnosis, (ie: Guttural pouch mycosisbrachial plexus avulsion,Otitis media, etc.) must treat that. 

Term
Transplacental transmission is a substantial and important source of infection to puppies for which parasite?

A - Ancylostoma caninum
B - Toxocara canis
C - Trichuris vulpis
D - Spirocerca lupi
E - Dipylidium caninum
Definition

Answer: B - Toxocara canis

In the pregnant female, dormant larvae of roundworms (Toxocara canis) mobilize and migrate into the developing fetus. Puppies as young as one week old may have roundworm larvae in their intestines.

Pups may also be infected via milk, because some roundworm larvae migrate to the mammary gland. Also, the immunity of the bitch to ascarid infection is partly suppressed during the perinatal period and large numbers of ascarid eggs can be passed in feces, contaminating the environment in which the puppies live. 

Hookworms (Ancylostoma caninum) are transmitted to puppies primarily via the colostrum and milk or later in life, from a contaminated environment. Georgi's Parasitology for Veterinarians says "Transplacental transmission, if it occurs at all, is overshadowed by transmammary transmission. A bitch exposed to only one substantial oral or percutaneous infection will shed A. caninum larvae in her milk for the next three lactations." 

WhipwormsTrichuris spp are acquired from the environment. 

Term
Which drug should not be administered to lactating dairy cattle?

A - Sulfamethazine
B - Pirlimycin
C - Thiabendazole
D - Oxytetracycline
E - Penicillin G
Definition

Answer: A - Sulfamethazine

Sulfamethazine cannot be used in lactating dairy cattle and there is no exception for extralabel use. Basically, almost all sulfonamides are banned from use in lactating dairy cows. 

Click here to see a list of drugs prohibited in food animals (No extra-label use).

Oxytetracycline, penicillin G, and Pirlimycin either have labeled indications for treatment of dairy cattle or can be used extralabel if warranted in the opinion of the attending veterinarian. 

Term
Most of a litter of European wild boar at a zoological park have died. The surviving piglets are in lateral recumbency with a frothy nasal discharge.

Necropsy of the piglets reveals pulmonary edema and copious fluid in the trachea and bronchi along with grayish- white necrotic foci on the myocardium.

The most likely diagnosis is:

A - Pseudorabies
B - Edema disease
C - Glasser’s Disease
D - Encephalomyelocarditis
E - Porcine Respiratory and Reproductive Syndrome
Definition

Answer: D - encephalomyelocarditis

Encephalomyocarditis virus (EMCV) is caused by a cardiovirus in the family picornaviridae. Confusingly, the virus is named for its effects on mice. 
Think of rodents and exotic zoo mammals with EMCV.

Pig-to-pig contact, contamination of swine feed and water by rodents or ingestion of dead rodents may cause disease. See pulmonary edema and copious transudate in the respiratory tract, causing cardiac failure.

Zoo outbreaks of EMCV have included lions, African elephants, rhinos, hippos, sloths, llamas, antelope and nonhuman primates. An outbreak of lion deaths at a Florida zoo in the USA occurred after feeding them the carcass of an African elephant that had died of EMCV. 

Edema disease is a neurologic disease caused by a hemolytic Escherichia coli producing Shiga toxin e2 and F18 pili resulting in high mortality in recently-weaned pigs. 

Glasser's Disease, caused by Hemophilus parasuis is usually an acute disease of 6 to 8 week-old pigs which causes fibrinous arthritis, polyserositis, and meningitis. 

Porcine Reproductive and Respiratory Syndrome (PRRS), is an arterivirus causing reproductive failure and post-weaning respiratory disease. 

Pseudorabies is a herpesvirus: infection causes CNS disease in neonates, respiratory disease in weaned pigs, and fever in all ages. 

Term
Damage to the left recurrent laryngeal nerve is associated with "roaring" in horses.

The left recurrent laryngeal nerve is a branch off of which cranial nerve?

A - Glossopharyngeal (CN 9)
B - Vagus (CN 10)
C - Trigeminal (CN 5)
D - Hypoglossal (CN 12)
E - Facial (CN 7)
Definition

Answer: B - Vagus

Damage to the recurrent laryngeal nerve (a branch of the Vagus (CN 10)) causes laryngeal hemiplegia - a paralysis of the abductor muscle (dorsal cricoarytenoid muscle) controlling the glottic cleft in the larynx. 

This allows the vocal fold (usually left side) to evert into the lumen of the larynx, obstructing airflow, leading to a roaring sound, and most importantly, slowing the horse.

Follow this link to see an endoscopic image of laryngeal hemiplegia.

If you have trouble remembering which cranial nerve is which, try this memory aid: 

1. On -Olfactory 
2. Old -Optic 
3. Olympus -Oculomotor 
4. Towering -Trochlear 
5. Tops -Trigeminal 
6. A -Abducens 
7. Fat -Facial 
8. Vested -Vestibulocochlear 
9. German -Glossopharyngeal 
10. Viewed -Vagus 
11. Some -Spinal accessory 
12. Hops-Hypoglossal 

Term
You have been called to investigate an outbreak of diarrheal disease of piglets which affected the healthiest animals in the herd, 1-2 weeks after weaning.

Some affected piglets had no signs except peracute death. Other affected piglets exhibit diarrhea, ataxia, paralysis, and recumbency.

What condition is at the top of your differential diagnosis list?

A - Clostridium perfringens type C enteritis
B - Epidemic transmissible gastroenteritis (TGE)
C - Edema disease
D - Porcine proliferative enteritis
E - Hemagglutinating encephalomyelitis virus (HEV)
Definition

Answer: C - Edema disease 

Edema disease is caused by entertoxigenic E. coli (ETEC). Look for severe acute illness ranging from peracute death with no signs to CNS involvement with ataxia, paralysis, and recumbency in healthiest pigs 1-2 weeks after weaning. 

Hemagglutinating encephalomyelitis virus (HEV), is almost exclusive to piglets less than 4 weeks old. Two clinical presentations: Vomiting and wasting disease (VWD) and encephalitic.

Clostridium perfringens type C enteritis ,also called enterotoxemia in other species is characterized by a HEMORRHAGIC diarrhea in 1-3 day old piglets. 

Porcine proliferative enteritis is principally a diarrheal disease of growing finishing (40- to 80-lb) pigs and young breeding pigs. 

Epidemic transmissible gastroenteritis (TGE) in non-immune pig herds characterized by HIGH MORBIDITY and HIGH MORTALITY in piglets less than 1 week old.

Term
What is the correct order of radiographic density of the following tissues, from least radiographically dense to most radiographically dense?

Urine-filled bladder
Humerus
Transarticular pin
Inflated lung
Abdominal fat

A - Humerus, Abdominal fat, Inflated lung, Transarticular pin, Urine-filled bladder,
B - Inflated lung, Abdominal fat, Urine-filled bladder, Humerus, Transarticular pin
C - Urine-filled bladder, Abdominal fat, Transarticular pin, Inflated lung, Humerus
D - Inflated lung, Urine-filled bladder, Humerus, Transarticular pin, Abdominal fat
E - Transarticular pin, Humerus, Urine-filled bladder, Abdominal fat, Inflated lung
Definition

Answer: B 

Radiographic density from least dense (black on x-ray) to most dense (white on x-ray) goes in this order: AIR-FAT-WATER-BONE-METAL.

Have a look at this radiograph of a dog who swallowed a metal coin to see example of differing radiographic densities. Depending on mineral density, a urolith in a urine-filled bladder may be more dense or less dense that the surrounding tissue. 

Here is a mnemonic to help you remember the 5 types of radiographic density:

1. Bubbles -Air (black)
2. Blubber -Fat (black)
3. Blood -Water, liquids (grey)
4. Bones -Bone and mineral (white)
5. Bullets -Metal (white) 

Term
A female veterinarian is pregnant and her physician tests her serologically for toxoplasmosis.

IgM is negative.
IgG is positive

What is the most appropriate interpretation?

A - Mother is safe, baby at risk
B - Both mother and baby at risk
C - Both mother and baby are safe
D - Need to re-check in 2 weeks for rising titers
E - Mother at risk, baby is safe
Definition

Answer: C - Both mother and baby are safe.

The toxoplasmosis organism causes birth defects in a developing fetus if a mother is infected for the first time in her life while pregnant (ie: IgM positive while pregnant). 


Toxoplasmosis is not generally dangerous to immune-competent people and a positive IgG result suggests an old infection. 

There are challenges to toxoplasmosis testing in pregnant women (false positives). If a pregnant woman is IgM positive, confirmatory tests must be done. 

Term
[image]
Definition

Answer: C - trichuris  (whipworms) 

Whipworms, (Trichuris spp) are typically found in the cecum and large intestine. Mainly in dogs, rare in cats. Trichuris suis in pigs can cause unthriftiness in younger animals. 

If clinical, look for signs of large bowel diarrhea (frequent urgent defecation of loose watery feces, possibly with mucus or fresh blood). Can be associated with a hypoadrenocorticism-like syndrome (hyponatremia, hyperkalemia, azotemia, metabolic acidosis). Whipworm infection has been suggested as one cause of cecocolic intussusception

Physaloptera spp (Stomach worms) may cause vomiting, anorexia, dark feces. 

Spirocerca lupi makes nodules in the esophageal, gastric, or aortic walls. Typically asymptomatic. 

Roundworms (Toxocara canis) may cause visceral and ocular larva migrans. 

Hookworms (Ancylostoma spp) may cause cutaneous larva migrans.

Term
Which parasite can cause cutaneous larva migrans in people?

A - Spirocerca sp.
B - Gasterophilus spp.
C - Habronema spp.
D - Ancylostoma spp.
E - Trichuris spp.
Definition

Answer: D - Ancylostoma spp (hookworms) 

Hookworms (Ancylostoma spp) may cause cutaneous larva migrans in people. Note thatRoundworms (Toxocara spp, Toxasacaris spp) are also zoonotic, causing visceral and ocularlarva migrans in people. 

In dogs, WhipwormsTrichuris spp are associated with a hypoadrenocorticism-like syndrome (hyponatremia, hyperkalemia, azotemia, metabolic acidosis). Whipworm infection has been suggested as one cause of cecocolic intussusception.

Habronema spp in horses can cause tumorlike stomach nodules and sometimes cutaneous lesions.

Term
During the necropsy of an 8 year-old mixed breed dog from the Southern United States, reactive granulomas in the esophagus containing bright red worms, 40 mm to 70 mm long are found.

What is the diagnosis?

A - Spirocerca lupi
B - Physaloptera spp
C - Gastrophilus spp
D - Ollulanus tricuspis
E - Haemonchus placei
Definition

Answer: A - Spirocera lupi

Typically asymptomatic, but large granuloma can cause esophageal obstruction. Large granomas may become neoplastic (osteosarcoma, fibrosarcoma). Some dogs develop spondylitis or enlargement of the extremities characteristic of hypertrophic osteopathy

All four of the other choices are gastric parasites. 
Ollulanus tricuspis is an uncommon gastric parasite of cats. 
Physaloptera spp is a stomach worm that may cause vomiting, anorexia, dark feces in dogs and cats. 
Haemonchus sppOstertagia spp and Trichostrongylus spp are found in the abomasum of ruminants. 
Gastrophilus spp are the larvae of horse bot flies, found in the stomach of horses.

Term
You examine a 4-year-old Standardbred mare in the Fall with a 5-day history of depression, left head tilt, facial paralysis and worsening gait. The horse is ataxic and knuckles on both forelegs, worse on the right.

There is muscle atrophy of the left shoulder and right hindquarters and strips of localized spontaneous sweating over the left trunk.

Which diagnostic test would best support the presumptive diagnosis?

A - Serum IgM capture ELISA for West Nile Virus
B - Plasma antibody test for St. Louis encephalitis
C - MRI to rule out nigropallidal encephalomalacia
D - Serum antibody titer for Eastern Equine Encephalomyelitis
E - Western Blot test of CSF for Sarcocystis neurona
Definition

Answer: E - Western blot for Sarcocystis neurona

Western Blot test on CSF for Sarcocystis neurona. Think of Equine protozoal myeloencephalitis (EPM) due to Sarcocystis neurona when you see ataxia and atrophy in horses, especially when neurologic signs are multifocal (ie:cranial (head tilt), spinal (foreleg ataxia). The muscular atrophy and localized strips of sweating are clues that EPM should be high on your DDX. 

Whenever you see a febrile neurologic horse, especially in the Fall, think also of the equine encephalidities, EEE, WEE and VEE as well as West Nile virus encephalitis. 

Nigropallidal encephalomalacia is the result of poisoning with yellow star thistle (image). Click here to see a table of Toxic North American range plants

Term
Which choice is the pastern joint?

A - Distal interphalangeal joint (PII-PIII)
B - Carpometacaral joint
C - Proximal interphalangeal joint (PI-PII)
D - Metacarpophalangeal joint
E - Intermetacarpal joint
Definition

Answer: C 

The proximal interphalangeal joint (PI-PII) is the pastern. Follow this link to see a fetlock and pastern anatomy image. Clinically important because one of the 3 most important nerve blocks, the foot block (also called pastern or abaxial sesamoid nerve block (ASNB)) is done just above the pastern.

Can use a pastern block to make a horse with acute laminitis comfortable. 

Term
How long after a booster rabies vaccination is a dog, cat or ferret considered to be currently vaccinated and protected against rabies?

A - Immediately
B - After 48 hours
C - After 7 days
D - After 14 days
E - After 24 hours
Definition

Answer: A - Immediately 

Immediately. According to the Compendium for Rabies Control, a peak rabies virus antibody titer is reached 28 days after initial vaccination and immediately after booster vaccination. 

Here are some thoughts on rabies: 
When in doubt, it is never wrong to check with your local health department. 

Basically all potential rabies exposures boil down to 2 questions: 
1. Who is involved?
Animal-Animal exposure (less alarm bells)
Animal bites/exposes human (more alarm bells)

2. Was animal vaccinated/up to date on vaccination?
Up to date on vaccs (less alarm bells, shorter observation)
Vaccinated, but not up to date (Handle on case-by-case basis)
Un-vaccinated pet (more alarm bells, euthanize or long observation period)
Wild animal, esp. bats, raccoon, skunk (euthanize, send head to state lab)

When dealing with rabies questions, ask yourself if this seems like a HIGH-risk exposure (ie: wild raccoon bites a child)
or a LOWER risk exposure (ie: Up-to-date vaccinated dog messes with woodchuck but no bite wounds on dog). 

For high risk lean towards euthanasia/testing or long quarantine. For low risk lean towards short observation period (10 days) and a rabies booster.

Term
What condition would be expected to have a positive Ortolani sign?

A - Coxofemoral luxation
B - Hip dysplasia
C - Osteochondrosis dissecans (shoulder)
D - Panosteitis
E - Cranial cruciate rupture
Definition

Answer: B - Hip dysplasia

A positive Ortolani sign indicates joint laxity, typically associated with hip dysplasia. A click/pop heard or felt by one hand pressed over coxofemoal joint while other hand presses upward and abducts knee. Think LARGE, hindlimb-LAME dogs, especially German shepherds. 

Follow these links to see radiographs of: early hip dysplasia,(no DJD yet), severe hip dysplasia (Note flattened, angular femoral head, poor congruence (parallel line) with acetabulum) and in contrast normal canine hips with deeply seated femoral heads and good congruence with acetabulae. 

Do a "Thumb test" for coxofemoral luxation (dislocated hip)- place thumb in the groove between greater trochanter & ischial tuberosity, externally rotate femur: thumb is pushed OUT of groove with normal hip, but stays in groove with luxated hip or fractured femoral head. 

A positive drawer sign indicates cranial cruciate rupture.

Panosteitis characterized by long bone pain on palpation and Hx of a shifting leg lameness in young, large dogs.

Term
What is a hygroma?

A - Inflammation of an acquired bursa over the dorsal carpus
B - Synovial membrane inflammation, dorsoproximal forelimb fetlock
C - Excessive granulation tissue around a healing wound
D - Saliva-filled cavity resulting from salivary duct trauma
E - A cyst located in the dorsolateral aspect of the nasal diverticulum
Definition

Answer: A 

hygroma is inflammation of an acquired bursa that develops where normally there is no bursa due to trauma. Common sites include the dorsum of the carpus in horses (or lateral elbow in large dogs). In horses, hygromas can be treated with rest, anti-inflammatory drugs, and bandaging. If this is not effective. drain and inject with corticosteroids or sclerosing agents. Infected hygromas require lavage, drainage, antimicrobials, and even resection but are often refractory.

A separate saliva-filled cavity resulting from trauma leading to extravasation of saliva from the duct is a salivary mucocele. Click here to see images of salivary mucocele, from the American College of Veterinary Surgeons (ACVS). 

Proud flesh is excessive growth of granulation tissue around a healing wound. 

A sebaceous cyst in the dorsolateral aspect of the nasal diverticulum (false nostril) is an atheroma. In humans, an atheroma usually refers to a fatty deposit in the intima (inner lining) of an artery, resulting from atherosclerosis. 

Term
Which choice describes inflammation of an acquired bursa over the dorsal carpus?

A - Villonodular synovitis
B - Atheroma
C - Ulceroproliferative faucitis
D - Suspensory desmitis
E - Hygroma
Definition

Answer: E - Hygroma 

hygroma is inflammation of an acquired bursa that develops where normally there is no bursa due to trauma to the dorsum of the carpus. 

Villonodular synovitis is an inflammation of the synovial membrane of the dorsoproximal aspect of the forelimb fetlock joints.

Ulceroproliferative stomatitis is a progressively worsening gingivitis and stomatitis in cats. The glossopalatine arches often have severely ulcerated, friable, inflamed, and proliferative lesions. Click here to see a cat with ulceroproliferative stomatitis

A sebaceous cyst in the dorsolateral aspect of the nasal diverticulum (false nostril) is an atheroma. In humans, an atheroma usually refers to a fatty deposit in the intima (inner lining) of an artery, resulting from atherosclerosis. 

Term
Which description correctly matches stringhalt?

A - Involuntary hyperflexion of the hock towards the abdomen
B - Thickening of the plantar tarsal ligament
C - Avulsion of the extensor process of the third phalanx
D - Effusion in the tarsal sheath of the deep digital flexor tendon
E - Progressive unilateral or bilateral hyperextension of the hindlimb
Definition

Answer: A 

Stringhalt is a myoclonic disease characterized by spasmodic hyperflexion of one or both hindlimbs. The presenting complaint is usually an involuntary hyperflexion of the hock towards the abdomen. Click here to see a video of a horse with classic stringhalt (classic symptoms start about 30 seconds into the video). Etiology is unknown. 

In contrast, progressive unilateral or bilateral hyperextension of the hindlimb is typically the presentation of spastic paresis (Elso heel) seen in cattle. Click here to see a cow with spastic paresis

Thickening or bowing of the plantar tarsal ligament is called curb.

Thoroughpin is distention/effusion/inflammation of the tarsal sheath of the deep digital flexor tendon proximal to the tarsus. 

Pyramidal disease describes avulsion or periostitis of the extensor process of the third phalanx. 

Term
Which description correctly matches gonitis?

A - Progressive unilateral or bilateral wide-angle glaucoma
B - Avulsion of the extensor process of the third phalanx
C - Infection of the crop and sinuses with Trichomonas gallinae
D - Inflammation of the stifle leading to degenerative joint disease
E - Inflammation of the mesorchium, parietal tunic and testicle
Definition

Answer: D 

Gonitis is an inflammation of the stifle leading to degenerative joint disease. 

In cases of glaucoma you might use gonioscopy to classify the glaucoma, detect iridocorneal outflow changes as the disease progresses and help determine the most appropriate medical and surgical treatments. 

Click here to see the classic big blue eye of acute glaucoma

Click here to see chronic glaucoma with lens luxation, a common sequella.

Term
Which of the following would be the best method for controlling anaplasmosis in adult cattle in the United States?

A - Vaccination with a modified live vaccine
B - Long-acting oxytetracycline for entire herd
C - Spray pastures with carbaryl insecticide
D - Serotest and cull affected cows
E - Imidocarb metaphylaxis
Definition

Answer: B - long acting oxytetracycline

Long-acting oxytetracycline for entire herd. Anaplasmosis is transmitted through tick vectors. Theoretically, tick control should eliminate the agent. Unfortunately, there is no effective method for controlling ticks on a range pasture. 

Affected cattle are long term carriers, so effective control must be based upon minimizing the number of inapparently infected cattle within the herd. This is best accomplished with 2 to 4 weekly injections of long-acting oxytetracycline.

Imidocarb has been used for treating carrier cows in some countries, but the drug is not currently approved for use in the United States in cattle.

Modified live vaccines exist, but have variable efficacy, availability and adverse effects.

Term
During a breeding soundness exam of a bull, preputial wash samples are typically cultured for which two organisms?

A - Toxoplasma gondii,Neosporosis
B - Ureaplasma diversum, Coxiella burnetii
C - Chlamydophila abortus, Leptospira hardjo
D - Brucella abortus,Mycoplasma bovis
E - Campylobacteriosis, Tritrichomonas (Trichomonas) foetus
Definition

Answer: E 

breeding soundness exam preputial wash samples may be cultured for two venereal diseases of cattle, bovine genital campylobacteriosis (Campylobacter fetus sp. venerealis) and bovine trichomoniasis (Tritrichomonas (Trichomonas) foetus). 

Both organisms can cause cause early embryonic loss in cattle and have similar presentations. 

Term
A 9 year old spayed female domestic shorthair cat is presented with a 5 month history of progressive right rear lameness. Apart from the worsening lameness, the cat acts normally and has a normal appetite.

T=101.3 F (38.5 C).....[N=100-103.1 F]
HR=120 bpm..............[N=100-140]
RR=36 brpm..............[N=16-40]

Physical examination reveals pain and swelling localized to the distal femur. Radiographs demonstrate a mixed lytic lesion in the distal femur that does not cross the joint space.

What is the most likely presumptive diagnosis?

A - Immune-mediate joint disease
B - Septic arthropathy
C - Anterior cruciate ligament tear
D - Osteosarcoma
E - Osteomyelitis
Definition

Answer: D - Osteosarcoma

Osteosarcoma characteristically creates boney lysis as well as boney production, and usually does not cross the joint space. Click here to see a radiograph of osteosarcoma

In general, osteosarcoma is less aggressive in cats than it is in dogs. In cats with appendicular osteosarcoma, amputation alone has a median survival time of more than 2 years. In contrast, median survival of dogs is only 4-5 months after amputation alone, and 10-12 months with amputation plus chemotherapy. 

Osteomyelitis most commonly involves fever, lethargy and cutaneous draining fistulous tracts. Osteomyelitis usually does cross the joint space. Click here to see a radiograph of osteomyelitis (distal PII-PII phalange). 

Anterior cruciate ligament tears localize in the the stifle joint causing instability ("drawer sign"), effusion, and degenerative changes. UNcommon in cats. 

Immune-mediated joint disease most commonly affects multiple joints. 

Septic arthropathy often is triggered by a penetrating bite or foreign body, leading to external swelling redness and inflammation or can be secondary to post-surgical contamination or hematogenous spread.

Though highly variable in its' presentation in the Southwest U.S. (desert), coccidioidomycosismay present with lameness in cats. Most commonly see systemic signs like fever, inappetence, weight loss and skin problems (draining skin lesions, SQ granulomatous masses, abscesses). 

Term
What is one cause of abdominal fat necrosis (lipomatosis) in cattle?

A - Fatty liver disease
B - Grazing tall fescue
C - Pregnancy toxemia
D - Chronic excess protein in ration
E - Aflatoxicosis
Definition

Answer: B - Grazing tall fescue 

One cause of abdominal fat necrosis (lipomatosis) in adult cattle (and some deer) is prolonged grazing of tall fescue infected with Acremonium coenophialum. Seen throughout the USA where tall fescue is the primary pasture grass. Over 90% of such pastures are infected with the endophyte.

Even without fescue exposure, hard masses of necrotic fat are relatively common in adult cattle. On rectal exam, the masses feel like "floating corks" similar to cotyledons, and may be mistaken for a developing pregnant uterus.

Remember the other fescue-related problem, fescue mycotoxins, which can cause lameness and hyperthermia in cattle and horses due to an ergot-like mold on tall fescue grass.

Fatty liver disease in cattle is a complex metabolic imbalance that can occur when an overconditioned cow reduces feed intake. Most common in periparturient cattle (think of fat cows at calving). 

Term
A 5-year old Arabian mare expels the placenta 2 hours after foaling.

On examination, it appears that the placenta has passed "inside out", with the allantoic side of the allantochorion exposed.

What is the correct interpretation?

A - Suggests premature placental separation
B - Foal at risk for peripartum asphyxia
C - Normal finding
D - Do not rebreed this mare
E - Sign of placental insufficiency
Definition

Answer: C - Normal finding

It is normal for a horse to expel the placenta "inside out", with the allantoic side of the allantochorion exposed. 

Following parturition, the placenta should be expelled within 3 hours, or it is considered retained in the horse. 

Term
Which two diseases would be on a differential diagnosis list for equine urinary incontinence?

A - Equine degenerative myelopathy, Botulism
B - Tetanus, West Nile virus encephalopathy
C - Nigropallidal encephalomalacia, Locoweed poisoning
D - Bracken fern toxicity, Equine encephalomyelitis
E - Cauda equina neuritis, Herpesvirus myeloencephalopathy
Definition

Answer: E 

Cauda equina neuritis and Herpesvirus myeloencephalopathy.

Horses with cauda equina neuritis (also called polyneuritis equi) have a progressive symetric LMN paresis of the tail, bladder, rectum, anal sphincter. Look for urinary incontinence, fecal retention and a weak or paralyzed tail. May see hind limb paresis if lumbosacral spinal cord is affected. 

Cranial nerves can also be affected, but typically cranial involvement is asymetric. May see temporal or masseter atrophy (Cranial Nerve 5), facial paralysis and exposure keratitis (Cranial nerve 7), head tilt or other CNS signs.
Cause is unknown, may be an autoimmune process. Grave prognosis. Eventually euthanized.

Herpesvirus myeloencephalopathy (EHV-1) may also present with urinary incontinence, but this is an uncommon manifestation of equine rhinopneumonitis. You would expect to hear a history of the more common EHV signs in other horses from the same farm, like respiratory disease ("snots") in foals and abortions in mares.

Bracken fern toxicity (Pteridium aquilinum) causes thiamine deficiency in monogastrics (like horses) and bone marrow depletion, aplastic anemia and bladder tumors in ruminants. In horses, look for signs of anorexia, weight loss, incoordination and a crouching stance with feet placed wide apart. Horses may have trembling muscles when forced to move.

Term
A 6-year old male intact Great Dane presents with a 2-week history of difficulty rising and hindlimb lameness. He has been intermittently anorexic.

The dog is febrile, and displays kyphosis (hunched back), stiff gait and spinal pain over the T3-L3 region with neurologic deficits. CBC shows a hematocrit of 38%..[N=37-55%], WBC=17,500..[N=500.4-15,300], U/A shows pyuria.

On a lateral spinal radiograph irregular lysis crossing the vertebral end plates is visible with collapse of at least one disc space, suggesting a diagnosis of diskospondylitis.

What test is indicated?

A - Chest radiographs, 3 views
B - Brucella canis titers
C - Myelography
D - Transtracheal wash
E - Fungal culture, Aspergillus
Definition

Answer: B - Brucella Canis titers 

Brucella canis titers are indicated. Diskospondylitis is a vertebral infection from hematogenous spread of bacteria (or fungi) from another site. Typically seen middle-aged big breeds, especially German Shepherds, Great Danes. 

Usually caused by hematogenous spread of STAPH AUREUS from primary infection elsewhere. See bacteremia, pyuria in 50%. Blood cultures positive in 75%. 

Diskospondylitis can be caused by Brucella canis, and sexually intact animals with suspected diskospondylitis should have Brucella titers measured. Recurrence is common with B. canis

Remember Brucella in horses with fistulous withers too- do serum agglutination titer forBrucella abortus, ESPECIALLY when horse has had contact with cattle. 

Term
Which disease of rabbits is reportable?

A - Myxomatosis
B - Rabbit (Shope) Fibroma Virus
C - Treponematosis
D - Encephalitozoonosis
E - Tularemia
Definition

Answer: E - Tularemia 

Tularemia is a disease primarily of wild rabbits caused by Francisella tularensis. Up to 90% ofhuman tularemia cases are linked to wild lagomorph exposure. Tularemia is a Category A bioterrorism agentREPORTABLE.

Treponematosis (rabbit syphillis, vent disease) is a venereal disease of rabbits caused byTreponema paraluis cuniculi. Click here to see a rabbit with cutaneous treponematosis

Myxomatosis ("big head") is a fatal disease of domesticated rabbits caused by myxoma virus, characterized by mucinous skin lesions or "myxomas". 

Encephalitozoonosis is a protozoal disease caused by Encephalitozoon (Nosema) cuniculi. Typically does not cause illness in rabbits, but is an emerging disease of immunosuppressed humans.

Term
A rabbit is presented with inflamed and chapped membranes of the anus and genital region. The genital area is scalded and raw, with brownish crusts and purulent exudate.

What two conditions top the differential diagnosis list?

A - Pasteurellosis, Ulcerative pododermatitis
B - Myxomatosis, Moist dermatitis
C - Treponematosis, Hutch burn
D - Tularemia, Cystitis
E - Glomerulonephritis, Coccidiosis
Definition

Answer: C - Treponematosis, Hutch burn 

Treponematosis (rabbit syphillis, vent disease) and hutch burn (urine burn) resemble each other and are often confused.

Treponematosis (rabbit syphillis, vent disease) is a venereal disease of rabbits caused byTreponema paraluis cuniculi. Affects the genitalia, may affect eyes and nose. Click here to see a rabbit with cutaneous treponematosis.

Hutch burn is caused by wet and dirty floors, affecting the anus and genitalia. Click here to see a rabbit with hutch burn.

Remember that cauda equina neuritis (polyneuritis equi) in horses may present with urine scald on the thighs. Other signs include a weak tail, hypotonic anus, urine dribbling and fecal retention. There may be a history of rubbing or chewing the tail head.

Term
[image]
Definition

In pigs, zinc deficiency causes parakeratosis. Zinc supplementation will resolve clinical signs.

Starter diets should contain 125 ppm zinc (and 0.9% calcium)

Grower diets should contain 75 ppm zinc (and 0.60-0.65% calcium)

Finisher diets should contain 50 ppm zinc (and 0.45 to 0.50% calcium) 

 

Parakeratosis may resemble exudative dermatitis ("greasy pig disease"), caused by Staphylococcus hyicus. Exudative dermatitis is more typically seen in younger suckling piglets and treated with antibiotics.

 

Sarcoptic mange (Sarcoptes scabiei var suis) is typically pruritic and treated with antiparasitics like ivermectin or pigs may be sprayed with lindane (0.05-0.1%) or malathion (0.05%). 

Term
[image]
Definition

Answer: E - Viral Isolation

This is the clinical picture of an outbreak of Highly Pathogenic Avian influenza (HPAI). AI viruses can be readily isolated from tracheal and cloacal swabs.

AI viruses vary in pathogenicity. In general, see nothing with subclinical infections. If clinical signs appear, may see sinusitis and respiratory signs in low pathogenicity strains, and fulminating multisystemic, hemorrhagic signs with highly pathogenic strains. Think-edema, cyanosis of head, wattle, and comb; hemorrhagic discoloration of feet and legs, and petechial hemorrhages on visceral organs on necropsy. Follow these links to see leg hemorrhages on a chicken with AI. 

The acute form of Newcastle disease must be differentiated from HPAI by isolation of a hemagglutinating virus identified by inhibition with Newcastle disease antiserum.

Term
A heifer has walked with an increasingly stiff and extended hock and stifle since 5 months of age. of life. The gastrocnemius muscle is contracted, and the animal walks with short pendulum-like steps. 

What is the most likely diagnosis? 

A - Spastic paresis
B - Femoral nerve paralysis
C - Vitamin E/Selenium deficiency
D - Coxofemoral luxation
E - Serous tarsitis
Definition

Answer: A - Spastic paresis 

This is the typical presentation of spastic paresis, also known as Elso heel. (Elso was a bull with many affected offspring). 

Look for an animal with a stiff hind leg or legs in full extension, whether walking or standing. Because spastic paresis is heritable, it is generally best to cull breeding animals, especially bulls. 

Term
[image]
Definition

This is ascites in a chicken. Suddenly dead or cyanotic, panting chickens with abdomens distended by ascites fluid suggests ascites syndrome (waterbelly). Essentially a form of right ventricular heart failure secondary to pulmonary hypertension or more rarely, hepatic failure. 

Because broilers affected with ascites syndrome frequently die on their backs, differential diagnosis includes flip-over disease.

Another cardiomyopathy to remember is round heart disease (spontaneous cardiomyopathy), but that is a disease of turkeys.

Dissecting aneurysm is another cause of sudden death, but animals display severe hemorrhage on necropsy. Click here to see a necropsied chicken with frank abdominal hemorrhage and a ruptured aorta

Term
[image]
Definition

Answer: E - Hypertrophic cardiomyopathy

This is an aortic thromboembolism, secondary to feline hypertrophic cardiomyopathy (HCM) the most common heart disease of cats. 

Physical signs of aortic thromboembolism include hindlimb paresis or paralysis, cyanotic nail beds, toe pads and cold extremities, decreased or absent pulses, contracted and painful hindlimb muscles, relentless crying.

Don't confuse this presentation with the nonpainful plantigrade stance of diabetic neuropathyin cats, an uncommon sequella of diabete mellitus.

Often, cats with HCM are asymptomatic and the problem is discovered on physical exam, with auscultation of a systolic murmur, evident in 80% of cats with HCM. Physical signs in cats with clinical HCM may include tachypnea, dyspnea, anorexia, vomiting and lethargy 

Term
[image]
Definition

Answer: D - Immunosuppression, anticoagulants and blood transfusion. This is the clinical picture ofImmune-mediated hemolytic anemia (IMHA). 


The basis of acute treatment includes:
1. Immunosuppressive therapy (Prednisone 1-2 mg/kg PO, SC or IM, q12h; or Dexamethasone Na phosphate, 0.15-0.25 mg/kg IV q12h)

2. Anticoagulant therapy (heparin)

3. Packed RBC transfusion is indicated if anemia is accompanied by tachycardia, tachypnea, bounding pulses, weakness.

Chronic therapy is based on immunosuppression with prednisone, for example, on tapering doses for 3-6 months, as long as hematocrit is stable.

Be aware that thrombocytopeniathromboembolism and disseminated intravascular coagulation (DIC) are three associated conditions to watch for in a dog with IMHA. 

Term
[image]
Definition

Answer: E - COLOSTRUM 

Ensure adequate colostrum uptake. The calf is head-pressing, one sign of meningitis due to central nervous system disease. 

Colisepticemic meningitis in calves is usually secondary to failure of passive transfer. The best prevention of colibacillosis is threefold:

1. Prevent failure of passive transfer (FPT) by ensuring colostrum uptake. 
2. Better sanitation. 
3. Vaccinate dam 6, 3 weeks with bacterin (K99 + E. coli) before calving. Because here are many variants of E.coli, vaccinating the dam to protect the calf is difficult. The cornerstone of prevention is to make sure the calf gets that colostrum

In adult cattle, thrombotic meningoencephalitis may be caused by Histophilus somni.

In pigs, see meningitis due to Haemophilus parasuis (Glasser's disease). Listeriosis causes a sporadic meningoencephalitis in cows and small ruminants

Term
[image]
Definition

Answer: C - Bone biopsy and culture 

Bone biopsy is required for definitive diagnosis of lytic bone lesion to differentiate between infection and neoplasia. Culture is indicated as well. 

The joint-crossing lytic damage seen between p2 and p3 is due to osteomyelitis (infection). Among bacterial causes, Staphylococcus spp are most common. 40%-50% of cases are polymicrobial with both anaerobes and aerobes present, and 70% of all cases test positive for some kind of anaerobe.

Osteosarcoma, an aggressive bone tumor, usually does not cross joints, but it still takes biopsy and culture to be definitive.

Term
[image]
Definition

Answer: A - Lupine spp.

Newborn calves with arthrogryposis ("crooked calf") have ankylosed, rigid limbs, scoliosis, kyphosis, and sometimes a cleft palate. The most common toxic cause of arthrogryposis in the calf or lamb is consumption of toxic alkaloids (anagyrine) in Lupine spp plants by pregnant dams. Click here to see a Lupine plant.

Adults that eat lupine may display inappetence, dyspnea, convulsions or death from respiratory paralysis. If lupines become infected with a fungus (Phomopsis leptostromiformis),mycotoxic lupinosis can cause hepatic damage.

Infectious causes of congenital arthrogryposis include in-utero infection with Bluetongue virusor Akabane virus.

Sorghum (Sudan grass, Johnson grass, Milo) can cause a neurologic toxicity, primarily in horses, or cyanide toxicity.

Term
[image]
Definition

Answer: D - Femoral head necrosis

This is avascular/aseptic necrosis of the femoral head (Legg-Perthes). Typically a young dog (4-11 mo.) problem of small and miniature breeds. Think of a Yorkie, Miniature poodle or Chihuahua presenting with a hind lameness and these radiographic findings. 

Manchester terriers have a genetic predisposition- do not breed affected animals. Unknown etiology. Typically Rx with NSAIDS, femoral head and neck osteotomy. 

Term
[image]
Definition

Answer: A - Skin scraping

Like most other species, South American camelids get sarcoptic mange (scabies). A skin scrape and microscopic exam can rapidly confirm the diagnosis. Treatment is an avermectin-family drug, SQ every 2 weeks. One report suggests amitraz is also effective. 

Term
[image]
Definition

Answer: B - Splints

This is a case of splints. Splints are periosteal reactions due to strain on the interosseous ligament between the third metacarpal or metatarsal (cannon) bones and either the second or fourth metacarpal/metatarsal (splint) bones.

Click here to see anatomic diagrams of splints, courtesy of the University of Missouri extension service.
In contrast to the exostosis and swelling of splints, here is a radiograph of a 4th metatarsal fracture

Trauma from concussion, strain from training, or conformation problems contribute to splint formation. Lameness is variable, usually disappearing as lesions become chronic. Ifsuspensory desmitis develops secondarily, prognosis is more guarded.

Bucked shins are a painful, acute periostitis on the cranial surface of the cannon bone. Click here to see a radiograph of bucked shins.

Epiphysitis (physitis) involves swelling around the long bone growth plates in young horses and can be a component of osteochondrosis.

Ringbone is an ostearthritis of the phalanges and pastern. Click here to see a radiograph of high ringbone.

Term
A 6-year old Standardbred jumper is presented with a 3-month history of poor performance and intermittent shifting hindleg lameness. On physical exam, there is poor muscling of the gluteal muscles and some asymmetry to the croup (rump).

The horse has a 7 cm. swelling over the hindquarters on the left dorsal side and shows pain and a reluctance to ventroflex the back when midline pressure is applied. On rectal palpation crepitation and shifting can be felt dorsally as the horse walks slowly forward.

What is the diagnosis?

A - Overlapping vertebral spinous processes
B - Croup myopathy (longissimus dorsi, supraspinous ligament)
C - Sacroiliac subluxation
D - Fibrotic ossifying myopathy
E - Coxofemoral luxation
Definition

Answer: Sacroiliac subluxation 

Think of sacroiliac subluxation or sprain in an athletic jumping horse with:
--Intermittent hindlimb lameness localized in the croup (rump)
--Evidence of back pain
--Swelling over the tuber sacrale (hunter's bumps)
--Crepitance felt rectally in the sacroiliac area. 

Horses with hunter's bumps can be sound if the injury has completely healed, but lameness can be observed if the injury is recent or has never resolved. Sometimes confused with stifle lameness.

Coxofemoral luxation is rare in horses due to a deep acetabulaum and the presence of the accessory ligament of the hip, unique to horses.

Overlapping vertebral spinous processes is a problem of thoracic and lumbar vertebra under the saddle area, seen in short-backed eventing horses (hunter/jumpers, dressage).

Back muscle and ligament strain is the #1 cause of back pain in horses, accounting for 33% of cases. Most commonly associated with the longissimus dorsi muscle, sublumbar illiopsoas muscles and the supraspinous ligament and also seen in eventing horses. 

Term
A client is considering the purchase of a 12-year old prize-winning thoroughbred mare for breeding. The pathology report from an endometrial biopsy says the mare is category III by the Kenney scoring system.

What advice is most appropriate to give the client?

A - Don't buy this mare for breeding
B - 80%-90% chance of bringing a foal to term
C - 10%-50% chance of bringing a foal to term
D - Best to breed by artificial insemination
E - 50%-80 chance of bringing a foal to term
Definition

Answer: A - Don't breed this mare for breeding 

Don't purchase a category III mare for breeding. Endometrial biopsy predicts a mare's ability to carry a foal to term and category III is the worst news. (Remember "3 strikes and you are out"). 

Endometrial biopsies are classified into 3 types by the Kenney system according to degree of periglandular uterine fibrosis and scarring. The Kenney system gives a % probability that a mare can carry a foal to term. Category I is best. Most horses fall in category II. Category III is worst. 

Category I No changes 
 (80%-90%)
Category IIA Minor changes 
 (50%-80%)
Category IIB Moderate changes 
 (10%-50%)
Category III Severe changes. 
 Less than 10% probability can carry a foal to term. 

Term
Several sheep are sick at a petting-zoo that has cows, horses, pigs, bison and white-tailed deer. One of the deer is also affected. Affected sheep are listless and off-feed, with serous or mucopurulent nasal discharge and rectal temperatures ranging from 105-107.5°F (40.5-42°C).

Physical exam shows swollen muzzles with edema and congestion of the lips, nose and face with small hemorrhages and ulcers on the mucous membranes. The ulcers appear where the teeth contact the swollen lips and tongue. Two affected sheep are lame.

What is the diagnosis?

A - Foot-and-mouth disease (FMD)
B - Contagious ecthyma (soremouth)
C - Peste des petits ruminants (PPR)
D - Bluetongue
E - Caprine arthritis encephalitis
Definition

Answer: D - Bluetongue

Swollen sore muzzles with mucous membrane erosions, high fevers and lameness suggests infection with Bluetongue virus. Bluetongue is almost exclusively seen in sheep, though white-tailed deer, pronghorn antelope and desert bighorn sheep in North America can be severely affected. Rare in cows. REPORTABLE.

Bluetongue is indistinguishable from Foot and Mouth disease (FMD), so that is a good second choice. But FMD is unlikely in the scenario above because FMD mainly affects pigs and cattle.

Term
A 4-year old thoroughbred mare is presented with a sudden onset of profuse, watery diarrhea and marked depression.

The mare has been intermittently treated with phenylbutazone over the last 2 months for minor lameness and was transported by trailer for 9 hours recently.

On physical exam the horse is dehydrated, with slow capillary refill time, purplish mucous membranes and cold extremities.

T=100.0 F (37.8 C)....[N=99-101.3 F]
HR=48 bpm...............[N=28-40]
RR=32 brpm..............[N=10-14]
PCV=65%.................[N=32%-53%]

The horse dies 3 hours later. On necropsy there is blood-stained fluid in the intestines and pronounced edema and hemorrhage in the wall of the large colon and cecum.

Which one of the following choices is the most likely diagnosis?

A - Nonsteroidal anti-inflammatory drug toxicosis
B - Sand enterocolopathy
C - Granulomatous enteritis
D - Parascaris equorum infestation
E - Colitis X
Definition

Answer: E - Colitis X 

This is the clinical picture of colitis X, an acute to peracute-onset of lethal diarrheal disease of horses. 

Look for shock signs, peracute severe diarrhea and a packed red cell volume (PCV) above 65%, and sudden death.

The cause is unknown but stress (like transport) or surgery often precedes disease.

This is a diagnosis of exclusion-if you rule out other peracute severe diarrheas (SalmonellosisPotomac horse fever (PHF), Clostridial enterocolitis, Lincomycin/Tetracycline toxicosis), you are left with presumptive colitis X.

Term
Several younger sheep in a large herd have died suddenly with a diagnosis of infectious necrotic hepatitis secondary to fluke infestation.

Which of the following is the most practical and effective control measure?

A - Treat ponds with copper sulfate molluscicide against lymnaeid snails
B - Clorsulon antihelmintic for whole herd
C - Clostridium hemolyticum bacterin for animals under 3 years
D - High-dose penicillin for clinical cases
E - Vaccinate all animals with Clostridium novyi toxoid
Definition

Answer: E - Vaccinate for Clostridium novyi toxoid

Active immunization with Clostridium novyi toxoid before the late summer is the most effective way to control and prevent Infectious necrotic hepatitis (Black disease) in sheep. 

Because vaccinated sheep have long-term immunity after only one shot, only new introductions to the flock (lambs and sheep brought in from other areas) need to be vaccinated. 

Reducing the number of snails (intermediate hosts for fluke cercaria) with molluscicides or by fencing off wet areas may not be practical due to expense or amount of pasture lost. Likewise, use of flukicides is complicated by the need for careful timing and long withdrawal times for meat and milk.

Term
A 4-day-old foal is presented with clinical signs of depression, icterus, head pressing, and disorientation. Serum biochemistry reveals increased levels of gamma-glutamyltransferase, alkaline phosphatase, and unconjugated bilirubin.

On further questioning, the owner mentions that he gave the foal an oral nutritional supplement when it was a day old but was not sure what was in it.

What could this supplement have contained to cause these clinical signs?

A - Iron
B - Vitamin E
C - Vitamin A
D - Copper
E - Selenium
Definition

Answer: A - Iron

Neonatal foals given oral iron supplements at birth can develop toxic injury to their liver and subsequent hepatic encephalopathy. 

Foals are born with a high serum iron level and absorb iron more readily than adult horses.

Term
Heat pasteurization of milk is done principally to prevent which one of the following organisms to people?

A - Mycobacterium paratuberculosis
B - Mycobacterium bovis
C - Enterotoxigenic Escherichia coli O157:H7
D - Mycobacterium avium
E - Mycobacterium tuberculosis
Definition

Answer: B - Mycobacterium bovis 

A century ago, Mycobacterium bovis (bovine tuberculosis) used to be a common zoonotic health threat, transmitted through milk to children through their emerging teeth at the gums.

Today, human cases of M. bovis are emerging more in binational communities with ties to Mexico who consume unpasteurized milk products (like fresh queso). 

Term
A person calls to to say she adopted a friend's dog. The former owner is a client at your clinic.

The new owner requests a copy of the dog's medical record and radiographs.

Which one of the following choices is the most appropriate action to take?

A - Give caller a copy of medical record and radiographs
B - Give a summarized record with personal identifiers (vet, owner names) blocked out
C - After physical exam, start a new medical record documenting previous issues
D - Give caller a copy of medical record only
E - Require original owner consent or court order to release information
Definition

Answer: E 

 

According to the AVMA's Principles of Veterinary Medical Ethics (2008 revision) 

"Ethically, the information within veterinary medical records is considered privileged and confidential. 

It must not be released except by court order or consent of the owner of the patient. 

Veterinarians should secure a written release to document that request." 

Term
[image]
Definition

Answer: B - Vitamin A Deficiency 

This is an aural abscess, secondary to vitamin A deficiency.

Captive terrestrial box turtles are most at risk, usually due to diets containing little vitamin A. 

Other presentations of hypovitaminosis A include froth from nose (a sign of respiratory disease) and renal disease. 

Term
A foal born 12 hours ago is observed swishing its' tail from side to side, straining with an arched back and rolling on the stall floor.

What test is indicated to rule in the presumptive diagnosis?

A - Digital exam of rectum
B - Cystocentesis
C - Endoscopy of gastric mucosa
D - Succussion of right caudal abdomen
E - Ultrasound of umbilicus/urachus
Definition

Answer: A - rectal exam 

Digital exam of rectum. Foals with meconium impaction will swish their tail from side to side, roll, and strain to defecate. A digital exam of the rectum will aid in rapid diagnosis.

Meconium impaction is the most common cause of colic in the newborn foal, typically seen within 24 hours of birth. More common in colts (males) than fillies because colts have a smaller pelvis.

Term
A 7-year old outdoor cat is presented with a month-long history of cough, dyspnea and weight loss. A heartworm antibody test is positive.

What is the most appropriate interpretation?

A - Cat does not have heartworm disease
B - Cat has heartworm disease
C - Presumptive feline asthma
D - A modified Knott's test is indicated
E - Further diagnostics needed
Definition

Answer: E - further diagnostics needed 

Further diagnostics needed. Diagnosis of heartworm disease (HW) in cats is difficult and requires an elevated index of suspicion and serial diagnostic tests. A positive antibody test in the cat only indicates EXPOSURE to heartworms, and further work up is indicated. 

A negative antibody is useful to rule-OUT HW disease in cats, but unfortunately, 14% of cats with HW infection may be antibody negative. 

A positive heartworm antigen test is diagnostic in cats, but a negative antigen doesn't say much because of high false negative antigen tests. 

Bottom line-there is no single test for HW in cats, and you need a full workup. Further diagnostics may include echocardiography, thoracic radiographs, a CBC and panel.

Term
[image]
Definition

Answer: C - Oxtetracycline

Oxytetracycline is an extremely effective treatment for Equine granulocytic ehrlichiosis(Anaplasma phagocytophilum), most commonly seen in California in the United States. Dexamethsone may help horses with severe ataxia and edema (20 mg, sid for 2-3 days).

Penicillin, chloramphenicol, and streptomycin are not effective. 

Term
A newborn foal with hypoxic ischemic encephalopathy and a poor suckle response is being treated supportively with mare's milk via nasogastric tube, IV fluids with 5% dextrose and broad spectrum antibiotics.

What other treatment is often critical to caring for foals like this?

A - Mannitol
B - Diazepam
C - Metaclopramide
D - Famotidine
E - Isoxuprine HCL
Definition

Answer: B - Diazepam 

Seizure control with diazepam (0.11-0.44 mg/kg) is critical for foals with hypoxic ischemic encephalopathy(HIE). Repeat dosage as needed. If long-term control is required, try phenobarbital (2-10 mg/kg, IV, bid-tid).

Dimethyl sulfoxide (DMSO 1 g/kg in a 10% solution, IV) can be used to help decrease cerebral edema.

Mannitol (1 g/kg as a 20% solution, IV) has been proposed to decrease cerebral edema, but must be used with caution: HIE foals often have subdural hemorrhage.

In addition to supportive care (warmth, nutrition, fluids, colostrum/plasma), remember the three "S's" of HIE problem management--
Sepsis
Seizures
Self-trauma 

Term
After diagnosis of appendicular osteosarcoma in a dog, the leg is amputated and chemotherapy is initiated. At follow-up monitoring exam 3 months later, thoracic radiographs are clear. What finding would suggest a guarded prognosis?

A - Depressed calcium-phosphorus ratio
B - Elevated albumin, depressed globulin
C - Elevated alkaline phosphatase
D - Polycythemia
E - Microcytosis
Definition

Answer: C - Elevated alkaline phosphatase

Elevated alkaline phosphatase is associated with a poor prognosis prior to surgery forosteosarcoma. If alkaline phosphatase stays elevated postoperatively, the prognosis is even more guarded. The median survival of dogs with appendicular osteosarcoma is only 4-5 months after amputation alone, and 10-12 months with amputation plus chemotherapy. 

90-95% of animals with osteosarcoma ultimately develop visible metastatic disease. The foundation of monitoring for osteosarcoma after amputation and adjunct chemotherapy is 3-view thoracic radiographs every 2-3 months. 

Click here to see a radiograph, metastatic lung tumors in a dog, and another radiograph offeline metastatic lung tumors

Think of microcytotic anemia with iron deficiency in young rapidly-growing animals like piglets and puppies and with portosystemic shunts. Think of polycythemia (high RBCS) most commonly secondary to dehydration

Term
A dog presents with a history of eating a shiny new United States penny that was hidden in a piece of birthday cake.

What problem may result?

A - Lead poisoning
B - Iron poisoning
C - Molybdenum poisoning
D - Nickel toxicity
E - Zinc toxicity
Definition

Answer: E - Zinc toxicity

Think of zinc toxicosis associated with ingestion of U.S. Lincoln pennies. All pennies minted since 1984 (and a few in 1983), are 97.5% zinc by weight.

Click here to see a radiograph of a dog who ate a penny.

Other sources of zinc include batteries, car parts, paint, zinc-oxide sunscreen creams, zippers, board-game pieces, screws and nuts on pet carriers, and the coating on galvanized metals like plumbing pipes and some cookware.

Copper toxicity is typically associated with copper containing drenches for sheep, or improperly formulated rations.

Molybdenum poisoning is an uncommon problem of cattle.

Iron poisoning is a sporadic problem of newborn piglets overdosed by iron injection. 

Term
[image]
Definition

Answer: B - Chlamydophila 

An inclusion body inside of a cell in eye discharge suggests Chlamydophila spp (ie:Chlamydophila felis). Follow this link to see the original Merck image of a chlamydial inclusion body.

Genetic testing led to taxonomic reclassification in 1999, but many sources still refer toChlamydophila felis infection as "Chlamydophila psittaci" or Chlamydiosis.

Although C. felis is a potential zoonotic disease, there are only a few reports of mild conjunctivitis in people from infected cats.

This is IN CONTRAST to the zoonotic potential of pet birds infected with Chlamydophila psittaci, which can apparently infect people more readily than the mammalian version. Single best ref on Psittacosis (what it is called when people get Chlamydophila), is the 2009 AVMA Compendium of measures to control Chlamydophila

Term
What is the most common clinical sign exhibited by a horse with cystic calculi (bladder stones)?

A - Straining to defecate
B - Uremic breath and urethral ulcers
C - Hematuria after exercise
D - Inappetance
E - Colic
Definition

Answer: C - Hematuria after exercise 

Bladder stones in horses are usually single, large spiculated stones composed of calcium carbonate. The most common clinical sign exhibited by horses with cystic calculi is hematuria after exercise. Hematuria is most evident toward the end of a voided urine stream. Other signs may include stranguria, pollakiuria and urinary incontinence. 

In adult male geldings, urethral obstruction may also be associated with cystitis. If there is urethral blockage by smaller uroliths, stranguria, pollakiuria, incontinence, restlessness, colic and stretched posture may be more prominent. 

Click here to see an endoscopic view of a urolith in a stallion and a necropsy image showing a nephrolith in the kidney of a horse.

Term
Which of the following diseases is characterized by schistocytes on a complete blood count?

A - Methemoglobinemia
B - Acetaminophen toxicity
C - Systemic lupus erythematosus
D - Cyanocobalamin deficiency
E - Disseminated intravascular coagulation
Definition

Answer: E - DIC 
 Disseminated intravascular coagulation. Schistocytes are produced by the mechanical fragmentation of erythrocytes on intravascular fibrin strands. They are most commonly associated with conditions that affect blood flow or clotting like disseminated intravascular coagulation(DIC), heartworm, hemangiosarcoma and glomerulonephritis


Cobalamin deficiency causes a macrocytic anemia (pernicious anemia).

Systemic lupus erythematosus may cause anemia, thrombocytopenia and leukopenia or leukocytosis.

Think of methemoglobinemia, heinz bodies and hepatotoxicity with acetaminophen toxicity in cats.

Term
A dog that was treated for vomiting with metaclopramide is now displaying involuntary muscle spasms and inappropriate aggression.

Which of the following choices is most appropriate to help reverse the adverse effects?

A - Yohimbine
B - Ace-promazine
C - Tiletamine (ketamine-diazepam combination)
D - Diphenhydramine hydrochloride
E - Ketamine
Definition

Answer: D - Diphenhydramine 

Diphenhydramine hydrochloride. Metoclopramide crosses the blood-brain barrier, where dopamine antagonism at the medullary chemoreceptor trigger zone (CTZ) causes an antiemetic effect. 

This dopamine antagonism can also cause adverse extrapyramidal signs, like involuntary muscle spasms, motor restlessness and inappropriate aggression. If recognized in time, extrapyramidal signs can be reversed by restoring the dopamine:acetylcholine balance with the anticholinergic action of an antihistamine like diphenhydramine hydrochloride.

The pyramidal and extrapyramidal systems are a complex series of upper motor neurons (UMN) that connect the cerebral cortex to distant body parts and influence muscular tone and control. 

The pyramidal system controls skilled muscle movement. 
The extrapyramidal system helps support the body against gravity (posture) and recruits spinal reflexes to initiate voluntary movement. 

Term
[image]
Definition

Answer: B - Ascaris 

Ascarids in pigs (roundworms) resemble ascarids in dogs and cats and also ascarids in horses. Ascarid eggs look a little like a fried egg with a big yolk. Click here to compare images:
Equine ascarid egg
Canine ascarid egg 
Porcine ascarid egg 

Here are images of several other key parasite eggs. 
Trichuris (whipworm)
Ancylostoma, Uncinaria (hookworm)
Tapeworm (resemble hookworm eggs, but larger) 
Strongyloides have small, thin-shelled (20-35 × 40-55 micrometers) embryonated eggs (larval worm visible inside). Strongyloides ova must be differentiated the embryonated ova oflungworms (Pigs, Metastrongylus, 33-42 × 51-63 micrometers). 

These are the key lungworms in different animals: 
Dictyocaulus in cattle, deer, donkeys and horses
Protostrongylus and Muellerius in sheep and goats
Metastrongylus in pigs
Oslerus (Filaroides) in dogs
Aelurostrongylus and Capillaria in cats.

Term
Which choice describes excessive growth of granulation tissue around a healing wound?

A - Ulceroproliferative faucitis
B - Villonodular synovitis
C - Hygroma
D - Suspensory desmitis
E - Proudflesh
Definition

Answer: E - Proudflesh

Proudflesh is excessive growth of cauliflower-like granulation tissue around a healing wound. Click here to see a link to case images of proud flesh and healthy granulation tissue on the leg of a horse.

hygroma is inflammation of an acquired bursa that develops where normally there is no bursa due to trauma to the dorsum of the carpus. 

Villonodular synovitis is an inflammation of the synovial membrane of the dorsoproximal aspect of the forelimb fetlock joints.

Ulceroproliferative faucitis is a progressively worsening gingivitis and stomatitis in cats. The glossopalatine arches (fauces) often have severely ulcerated, friable, inflamed, and proliferative lesions. Click here to see a cat with ulceroproliferative faucitis

Term
A valuable female show dog is approaching estrus and the owner wants her bred by artificial insemination to one of the most successful and expensive stud dogs in the country.

What testing method is most accurate to insure optimum timing for breeding the bitch?

A - LH monitoring
B - Vaginal cytology
C - Progesterone assays
D - Vaginoscopy
E - Estrogen levels
Definition

Answer: A - LH monitoring 

Daily testing for the LH surge is the most accurate method of ovulation timing. The day of the LH surge becomes "day 0" and you plan breedings from there, typically at days 2, 4 and 6. You might choose LH monitoring for frozen or chilled semen breedings, infertility cases or breedings with valuable, or subfertile studs. 

Progesterone assays, vaginal cytology and vaginoscopy are all useful adjuncts to breeding management. Progesterone starts to increase at about the same time as the LH surge- although progestrone levels are not as accurate as measuring the actual LH surge, the tests may be more convenient and available than LH tests. 

Estrogen measurement is of little value for ovulation timing because peak levels vary from dog to dog and do not necessarily correlate with the fertile period. Increased estrogen speeds up the turnover of vaginal epithelial cells, which causes progressive cornification seen on vaginal cytology.

Term
The Animal Welfare Act is enforced by which U.S. government agency?

A - National Institute of Health (NIH)
B - Centers for Disease Control and Prevention (CDC)
C - Food and Drug Administration (FDA)
D - Department of Agriculture (USDA)
E - Institutional Animal Care and Use Committee (IACUC)
Definition

Answer: D - USDA 

The United States Department of Agriculture (USDA) enforces the Animal Welfare Act

Specifically, it is the Animal and Plant Health Inspection Service (APHIS) within the USDA that is responsible.

The Animal Welfare act sets minimal standards for care of laboratory animals including dogs, cats and non-human primates. It regulates the pet trade, animal transportation and exhibition and licensure of animal dealers. Since 1976 it has prohibited most forms of commercial animal fighting. 

Term
A multiparous cow that calved two months ago is presented with a history of blood-stained urine for the last week. Today, the cow is off-feed and colicky, switching her tail restlessly and urinating frequently.

T=103.8 F (39.9 C)....[N=101.5-103.5 F]
HR=70 bpm...............[N=55-80]
RR=32 brpm..............[N=10-30]

A cowside dipstick test shows hematuria and proteinuria. The bladder, ureters and left kidney feel grossly normal on rectal exam.

Which of the following choices is the most likely diagnosis?

A - Pyometra
B - Dioctophyma renale
C - Leptospirosis
D - Polycystic kidney disease
E - Pyelonephritis
Definition

Answer: E - Pyelonephritis 

This is the clinical picture of a cow with early signs of pyelonephritis. Onset is typically insidious. The first clinical sign may be passage of blood-stained urine (hematuria) in an otherwise normal animal. 

As infection (usually Corynebacterium spp or E. coli) ascends up the ureters into the kidney, the cow may display frequent attempts to urinate, anorexia, a slight fever, loss of production, colic, restlessness, tail switching, polyuria, more hematuria or pyuria. 

Early in the disease, enlarged ureters and involvement of the kidney may not be detectable on rectal palpation. In chronic cases, the left kidney may be enlarged, with loss of lobulation and pain. The cow may show colic, diarrhea, polyuria, polydipsia, stranguria, and anemia. 

Term
A 7-year old dog weighing 24 lbs (10.9 kg) is presented after the owner caught the dog eating warfarin-containing rat poison the day before.

The dog has no overt clinical signs at presentation. Baseline coagulation parameters are normal (Prothrombin time (PT), Activated partial thromboplastin time (aPTT) and Proteins induced by vitamin K1 absence or antagonism (PIVKA).

The dog is confined to limit exercise and monitored. At 72 hours PT and PIVKA tests are elevated.

Which of the following is the most appropriate treatment step?

A - Oral vitamin K1, SID
B - Plasma transfusion
C - Activated charcoal
D - Injectable vitamin K1, EOD
E - Blood transfusion
Definition

Answer: A - oral vitamin K 

Oral vitamin K1. If PT or PIVKA tests elevate at 48 or 72 hours in a case-patient with suspectedanticoagulant rodenticide toxicity, start oral vitamin K1 at 1.5-2.5 mg/kg q 12h or 3-5 mg/kg once daily. 

Continue treatment for 14 to 30 days, depending on the specific anticoagulant. Warfarin is a first generation, shorter-acting anticoagulant, brodifacuom is a second-generation long-acting anticoagulant. 

PT or PIVKA should be checked 48 hours after the last dose of vitamin K1. If PT or PIVKA is still elevated 48 hours after last dose (16 to 32 days postexposure), restart vitamin K1 treatment for 1 week, then recheck PT or PIVKA 48 hours after last dose. Once the PT or PIVKA is normal at 48 hours, you can stop vitamin K1 treatment.

Injectable vitamin K1 has no advantage over oral vitamin K1 and can cause anaphylaxis in some animals, even when injected subcutaneously. A blood or plasma transfusion is indicated for animals that present with overt clinical signs (hemorrhage. dyspneic, pale).

Term
A ten year old neutered male cat is presented with an acute onset of vomiting and anorexia. On physical examination, the patient is dehydrated with a painful abdomen.

Bloodwork:
ALB=2.2 g/dL.......[N=2.6-4.0], ALK PHOS=88 U/L..[N=3-65]
ALT=112 U/L........[N=8.2-57], AMYL=2483 U/L..[N=270-1,462]
BUN=42 mg/dL.....[N=10-30], KK=3.3..[N=3.5-5.1]
CL=108 mEq/L......[N=109-122], WBC=22,200..[N=3800-19,500]
LYMPHS=1404/uL..[N=1000-4800], NEUTS=15538/uL..[N=0-3000]
Ultrasound: Mass effect on the region of the pancreas
Radiographs: Decreased serosal detail in the cranial abdomen

In addition to supportive care (IV fluids, anti-emetics) and pain control, which of the following treatment options is indicated?

A - Pancreatic enzyme replacement therapy
B - Hypoallergenic diet
C - Tylosin; Cisapride
D - Prednisone
E - Trimethoprim-sulfamethoxazole
Definition

Answer: D - Prednisone

Prednisolone is a viable treatment option for some patients and may also mitigate associated diseases such as cholangiohepatitis and inflammatory bowel disease. 

The mainstay of pancreatitis therapy is supportive care, with IV fluids, pain control (meperidine, butorphanol, fentanyl, buprenorphine) and antiemetics. If vomiting can be controlled, judicious offering of low-fat food is indicated in cats.

Prednisolone is contraindicated in feline diabetes mellitus and canine pancreatitis. Antibiotics are only indicated in cases where an underlying cause can be identified that requires their use.

Term
When a mare is being treated with erythromycin and she is nursing a foal, the foal is at high risk of developing enterocolitis due to which of the following organisms?

A - Rhodococcus equi
B - Clostridium novyi
C - Lawsonia intracellularis
D - Escherechia coli
E - Clostridium difficile
Definition

Answer:E -  C. Diff

 

A history of recent antimicrobial therapy is common in cases of Clostridium difficileassociated diarrhea. Adult horses exposed to erythromycin are particularly at risk for C. difficile enterocolitis.

Clostridium novyi is the cause of infectious necrotic hepatitis, which is primarily seen in sheep but can also be seen in cattle, hogs, and horses.

Lawsonia intracellularis is the cause of proliferative enteropathy, resulting in diarrhea and hypoproteinemia in foals and swine.

Rhodococcus equi is a notable cause of pneumonia in older foals characterized by pulmonary abscessation as well as some extrapulmonary manifestations.

Escherechia coli can be a cause of septicemia and diarrhea in foals and calves.

Term
An adult horse with clinical signs of voluminous gastric reflux, depression, colic, and fever is most likely suffering from which one of the following choices?

A - Proximal enteritis
B - Cantharidin toxicity
C - Right dorsal colitis
D - Proliferative enteropathy
E - Ulcerative duodenitis
Definition

Answer: A - Proximal enteritis 

Proximal enteritis, or duodenitis-proximal jejunitis, is a clinical syndrome characterized by large volumes of gastric reflux resulting from excessive fluid and electrolyte secretion into the small intestine and small intestinal inflammation and edema. 

Laminitis is an important potential sequela. The cause is unknown but several bacteria and toxins including Clostridium difficileClostridium perfringens, Salmonella, and fumonosin B1 mycotoxins have been implicated.

Ulcerative duodenitis is a disorder of foals resulting in fever, colic, diarrhea, and delayed gastric emptying. Lawsonia intracellularis causes proliferative enteropathy in foals and weanlings, a disease characterized by hypoproteinemia, diarrhea, chronic ill thrift, and ventral edema.

Right dorsal colitis is typically a result of non-steroidal anti-inflammatory drug toxicity and results in hypoproteinemia and colic. 

Cantharidin toxicity (blister beetles, Epicauta spp.) causes a wide range of clinical signs predominated by profuse diarrhea, stranguria and pollakiuria, and colic.

Term
[image]
Definition

Answer: C - Actinobacillus Pleuropneumoniae

This is a classic scenario of an outbreak of pleuropneumonia, caused by Actinobacillus pleuropneumoniae.

Term
During a routine immunization visit for a 2 year-old neutered male Newfoundland dog, a systolic murmur is detected which is heard loudest on the left side of the chest between the 2nd and 5th intercostal (IC) space and at the thoracic inlet lateral to the trachea.

Which condition is highest on a differential diagnosis list?

A - Aortic stenosis
B - Tricuspid dysplasia
C - Pulmonic stenosis
D - Patent ductus arteriosus (PDA)
E - Mitral dysplasia
Definition

Answer: A) Aortic stenosis 

Aortic stenosis (also called sub-aortic stenosis [SAS]), is a systolic murmur which may be heard most loudly on the left chest between the 2nd and 5th intercostal (IC) space or at the thoracic inlet (lateral to trachea). 

Inherited in Newfoundlands. Predilection in many BIG BREEDS- German Shepherd, Golden Retriever, Boxer, Rottweiler. 

Mitral dysplasia and other mitral valve problems are heard further back on left at 5th-6th IC. More common in CATS. 

With Pulmonic stenosis see RIGHT ventricular hypertrophy, because pulmonic valves blocks outflow from R ventricle (mostly dogs). Follow this link to see a Merck image of Rt. ventricular hypertrophy. 

Tricuspid dysplasia is heard further back on RIGHT at 5th-6th IC. Uncommon. 

Expect a continuous murmur with Patent ductus arteriosus (PDA). Vast majority detected at first vaccination visit. 

Term
A 7 year-old intact female poodle is presented. The owner has noted a lot of drinking and some urinary "accidents" in the house in the last week and that the dog "hasn't seemed herself" either.

Yesterday she vomited twice. On physical exam the dog appears depressed and listless, has dark injected mucous membranes and a distended tender abdomen.

T=104 F (40 C)..[N=99.5-102.5 F]
HR=100 bpm.....[N=110-120]
RR=30 brpm......[N=15-34]

What diagnosis is of most immediate concern?

A - Hepato-splenic tumor
B - Pregnancy
C - Diabetes mellitus
D - Intestinal foreign body
E - Pyometra
Definition

Answer: E - Pyometra

The preferred answer is Canine pyometra

Pyometra is a life-threatening emergency which typically requires immediate surgery (ovariohysterectomy) if the cervix is closed and there is no obvious discharge. Open cervix pyometras are likely to present with a purulent discharge, simplifying Dx, but surgical ovariohysterectomy is still STRONGLY recommended.

Pyometra should always be near the top of your DDX when presented with an older intact female dog or cat with a fever and distended abdomen

Term
Which one of the following drugs is used primarily to treat pituitary-dependent hyperadrenocorticism (Cushing's disease) in dogs?

A - Liothyronine
B - Mitotane (o,p DDD)
C - Fludrocortisone acetate
D - Phenoxybenzamine
E - Imidocarb
Definition

Answer: B - MItotane (o, p DDD)

Mitotane (o,p DDD) is used to treat pituitary-dependent hyperadrenocorticism (Cushing's disease). Mitotane selectively destroys the glucocorticoid-secreting cells of the adrenal cortex.

REMEMBER dogs undergoing mitotane therapy should receive supplementary glucocorticoids when undergoing stress (ie: surgery, trauma, acute illness). 

Alternative treatments include l-Deprenyl (decreases pituitary ACTH secretion) or Ketoconazole (inhibits enzymes of cortisol synthesis, may be used in dogs who do not tolerate Mitotane).

NOTE Trilostane, a drug is also used in treatment of hyperadrenocorticism. Survival times appear to be similar between Trilostane and Mitotane. 

Trilostane is typically given SID to BID lifelong, while maintenance Mitotane is typically given 2-3 times per week. Long-term, Trilostane is more expensive than Mitotane. 

Trilostane appears to be safer in general but patients must be monitored for adrenal necrosis, which can lead to Addisonian crisis and death if unrecognized. Experienced practitioners may disagree on whether to use Trilostane vs. Mitotane. 

Term
[image]
Definition

Answer: E - Estrus 

Think CORN FLAKES with ESTRUS. Look for greater than 90% CORNIFIED superficial cells (angular, sharp edges like , tiny pyknotic nuclei or no nuclei) when you think canine estrus, and NO NEUTS. Follow this link to see the an estrus vaginal cytology image.

In contrast, DI-estrus vaginal smears will have more than 10% nucleated, round-edged parabasal and intermediate cells. Look for the reappearance of some neutrophils (variable numbers). Follow this link to see a DI-estrus vaginal cytology image

Full cornification usually coincides with receptivity. Serum progesterone can predict the LH surge and help estimate ovulation. 

RULE OF THUMB: Breed bitch as soon as she will allow a male to mate or as soon as you see greater than 90% cornified superficial epithelial cells in a vaginal smear. Typically breed every 2-4 days until bitch enters diestrus for maximum fertility success

Term
A 7-week old male Yorkshire terrier is presented with a 2 week history of on and off vomiting and diarrhea that began around the time he was weaned.

The owners relate that he seems to "drink and pee a lot". They report pacing, disorientation, weakness, and "stumbling around".

Physical exam is unremarkable, but only one testicle has descended. As the puppy explores the room he appears ataxic, stumbles a few times, and bumps his head into the wall.

What is the clinical diagnosis?

A - Canine distemper
B - Diabetes insipidus
C - Portosystemic shunt
D - Lead poisoning
E - Congenital hiatal hernia
Definition

Answer: C - portosystemic shunt

Signs of hepatic encephalopathy (ataxia, disorientation, vomiting, diarrhea) beginning after weaning combined with polyuria/polydipsia ("drink and pee alot") in a cryptorchid male Yorkshire terrier says Congenital Portosystemic Shunt

Seen most in pure-breeds. Think SMALL Maltese, Yorkshire terrier, Min. Schnauzer. (But can see in Old English sheepdog, Irish Wolfhound) 

Usually in YOUNG animals, especially after weaning. 50% of males are CRYPTORCHID.

Lead poisoning can cause vomiting, diarrhea and CNS signs (blind, hyperactive, seizures). Can see PU/PD in older animals but Hx here puts a shunt first on DDX. 

Term
[image]
Definition

Answer: A - Osteosarcoma 

This is a classic image of Osteosarcoma, a COMMON, aggressive bone tumor typically found in the appendicular skeleton, especially distal radius.

90% have microscopic metastases to lungs by time of Dx, (but less than 10% will have visible thoracic metastasis at time of diagnosis).

Usually, does not cross joint (unlike osteomyelitis, which usually DOES cross joint). Look for soft tissue swelling, periosteal proliferation, sunburst periosteal reaction (33%), possible pathologic fractures.

Term

An obese 6 year-old neutered male Pomeranian presents with a 2-year history of intermittent dry, honking cough. This morning he started retching. The dog never boards in a kennel and has not traveled recently.

On physical exam, a cough can be elicited by pressing on the trachea. A heart murmur is noted, loudest on the left at the 5th-6th intercostal space. The rest of the exam was unremarkable.

T=102.2 F (39 C)..[N=99.5-102.5 F]
HR=120 bpm .......[N=80-120]
RR=80 brpm .......[N=15-34]
CRT=2 sec...........[N= less than 2 sec], MM=pink

What diagnosis is at the top of the differential list? 

A - Tracheal obstruction
B - Chronic bronchitis
C - Tracheal collapse
D - Tracheobronchitis
E - Congestive heart disease

Definition

Answer: C - Tracheal collapse 

When you hear a "HONKING COUGH" and heart murmur in a fat little dog that coughs after touching his tracheal area, think tracheal collapse. Toy/Min. breeds (Pomeranians, Toy poodle, Min. poodle, Yorkies) are predisposed.

Mitral insufficiency / dysplasia murmurs are often detected concurrently.

Intermittent hx and lack of fever suggest a chronic, non-infectious cause. For definitive diagnosis do thoracic radiographs, bronchoscopy.

Term
A 7-year old male German shepherd presents with a history of weakness in the hind limbs, urinary incontinence and recent obsessive chewing around his tail area. Dorsiflexion of the tail over the back and extension of the hind limbs elicits a painful response.

There is moderate hindlimb ataxia. He does not withdraw each hind leg when a toe is pinched, but bears weight on the hindlimbs. Patellar reflexes are normal.

What is the clinical diagnosis?

A - Diskospondylitis
B - Hip dysplasia
C - Wobbler syndrome
D - Radiculoneuritis
E - Cauda equina syndrome
Definition

Answer: E - Cauda equina syndrome 

This is a common presentation of Cauda equina syndrome. Look for PAIN in the lumbosacral area (elicited by tail raise, hindlimb extension), LMN hindlimbs, especially Sciatic nerve damage at L7-S1 (lack withdrawal), +/- urinary/fecal incontinence, +/- SELF-MUTILATION of tail, perineum, pelvic limb. 

Lesion due to compression of cauda equina at L7-S1 (lumbosacral stenosis). Can be congenital (abn development dorsal arch L7-S1, small dogs, Border Collies) or acquired (degenerative changes, big dogs, especially German Shepard, Rottweiler, Boxer). 

Wobbler syndrome is a cervical spinal cord disease (also called cervical vertebral instability/ caudal cervical spondylomyelopathy). In adult form it is an intervertebral disc disease, usually seen at C5-C6 or C6-C7 in Dobies> 5 yrs, Great Danes> 2 years. 

Diskospondylitis is a good second choice on your DDX of a large middle aged dog presenting with lumbosacral pain. Due to bacterial/fungal infection of intervertebral disk and adjacent vertebral bodies. Look for systemic signs like fever (1/3 of patients), weight loss, anorexia. Follow this link to see a Merck image of diskospondylitis

Term
A dog hit by a car presents with stiff hypermetric forelimbs and flaccid, paralyzed hindlimbs.

Where is the lesion likely to be?

A - Cranial cervical: C1-C5
B - Lumbosacral L4-S3
C - Cervicothoracic: C6-T2
D - Thoracolumbar T3-L3
E - Cannot tell without more information
Definition

Answer: D - T3-L3

Thoracolumbar T3-L3 . This is likely to be Schiff-Sherrington syndrome ie: severe spinal cord trauma T3-L3, with thoracic limb extensor rigidity and hind limb flaccid paralysis. Lesion is caudal to T2, but inhibitory neurons in lumbar spinal cord (especially L2-L4) affect neurons in the cervical intumescence ( C6-T2). 

With severe trauma T2-T13, inhibitory pathways are interrupted; Cervical intumescence neurons are "released" and cause extensor hypertonia in the forelimbs.

Can localize by checking cutaneous trunci reflex-The lesion is usually 1-2 vertebrae cranial to the line of analgesia (where dog does not feel pinching skin).

Term
[image]
Definition

Answer: C - Wobblers 

The presentation, age and breed suggest this is most likely to be Wobbler syndrome (also called cervical vertebral instability/ caudal cervical spondylomyelopathy). In the adult form it is an intervertebral disc disease, usually seen between C5-C6 or C6-C7 in Dobermans more than 5 yrs old and Great Danes more than 2 years old. 

Test of choice is MRI. At least need myelography to Dx exact location and type of spinal cord compression. 

Follow this link to see a Merck image of Wobbler syndrome. Note how contrast media disappears as it approaches caudal cervical vertebra. 

Rx is steroids and surgery. 

Term
[image]
Definition

Answer: A - Mycoplasma haemofelis

This is the anemia of Mycoplasma haemofelis (formerly Haemobartonella felis). Once thought to be a rickettsia, it is now known to be a form of Mycoplasma bacteria (called Eperythrozoon felis in Europe/Australia). See cocci, rings on edges of RBCs for (not always easy to see), and note signs of regenerative anemia: polychromasia of RBCs (variable color), presence of nucleated RBCs. Basically an immune-mediated anemia, with body removing parasitized RBCs from circulation.

Heinz body anemia is due to oxidant injury to RBCs-Think Acetaminophen toxicity in cats. Heinz bodies are precipitated hemoglobin.

Cytauxzoonosis is a fatal protozoal anemia of the SOUTHERN U.S. (Texas to Florida).

Babesiosis is more a dog dusease (transmitted via brown dog tick, Rhipicephalus sanguineus) and seen in SOUTHWEST (W. Texas to S. California).

Term
A rabbit is presented with inflamed and chapped membranes of the anus and genital region. The genital area is scalded and raw, with brownish crusts and purulent exudate.

What two conditions top the differential diagnosis list?

A - Glomerulonephritis, Coccidiosis
B - Treponematosis, Hutch burn
C - Myxomatosis, Moist dermatitis
D - Tularemia, Cystitis
E - Pasteurellosis, Ulcerative pododermatitis
Definition

Answer: B - Hutch burn 

Treponematosis (rabbit syphillis, vent disease) and hutch burn (urine burn) resemble each other and are often confused.

Treponematosis (rabbit syphillis, vent disease) is a venereal disease of rabbits caused byTreponema paraluis cuniculi. Affects the genitalia, may affect eyes and nose. Click here to see a rabbit with cutaneous treponematosis.

Hutch burn is caused by wet and dirty floors, affecting the anus and genitalia. Click here to see a rabbit with hutch burn.

Remember that cauda equina neuritis (polyneuritis equi) in horses may present with urine scald on the thighs. Other signs include a weak tail, hypotonic anus, urine dribbling and fecal retention. There may be a history of rubbing or chewing the tail head.

Term
[image]
Definition

Answer: E - Supplemental Zinc 

 

In pigs, zinc deficiency causes parakeratosis. Zinc supplementation will resolve clinical signs.
Starter diets should contain 125 ppm zinc (and 0.9% calcium)
Grower diets should contain 75 ppm zinc (and 0.60-0.65% calcium)
Finisher diets should contain 50 ppm zinc (and 0.45 to 0.50% calcium) 

Parakeratosis may resemble exudative dermatitis ("greasy pig disease"), caused byStaphylococcus hyicus. Exudative dermatitis is more typically seen in younger suckling piglets and treated with antibiotics.

Sarcoptic mange (Sarcoptes scabiei var suis) is typically pruritic and treated with antiparasitics like ivermectin or pigs may be sprayed with lindane (0.05-0.1%) or malathion (0.05%). 

Term
A 9 year old spayed female domestic shorthair cat is presented with a 5 month history of progressive right rear lameness. Apart from the worsening lameness, the cat acts normally and has a normal appetite.

T=101.3 F (38.5 C)..[N=100-103.1 F]
HR=120 bpm...........[N=100-140]
RR=36 brpm...........[N=16-40]

Physical examination reveals pain and swelling localized to the distal femur. Radiographs demonstrate a mixed lytic lesion in the distal femur that does not cross the joint space.

What is the most likely presumptive diagnosis?
A - Osteomyelitis
B - Immune-mediate joint disease
C - Anterior cruciate ligament tear
D - Septic arthropathy
E - Osteosarcoma
Definition

Answer: E - Osteosarcoma 

Osteosarcoma characteristically creates boney lysis as well as boney production, and usually does not cross the joint space. Click here to see a radiograph of osteosarcoma

In general, osteosarcoma is less aggressive in cats than it is in dogs. In cats with appendicular osteosarcoma, amputation alone has a median survival time of more than 2 years. In contrast, median survival of dogs is only 4-5 months after amputation alone, and 10-12 months with amputation plus chemotherapy. 

Osteomyelitis most commonly involves fever, lethargy and cutaneous draining fistulous tracts. Osteomyelitis usually does cross the joint space. Click here to see a radiograph of osteomyelitis (distal PII-PII phalange). 

Anterior cruciate ligament tears localize in the the stifle joint causing instability ("drawer sign"), effusion, and degenerative changes. UNcommon in cats. 

Immune-mediated joint disease most commonly affects multiple joints. 

Septic arthropathy often is triggered by a penetrating bite or foreign body, leading to external swelling redness and inflammation or can be secondary to post-surgical contamination or hematogenous spread.

Though highly variable in its' presentation in the Southwest U.S. (desert), coccidioidomycosismay present with lameness in cats. Most commonly see systemic signs like fever, inappetence, weight loss and skin problems (draining skin lesions, SQ granulomatous masses, abscesses). 

Term
[image]
Definition

Answer: E - Distal humerus condylar fracture

Fractures of the distal humerus are the most common fracture types seen in potbellied pigs, usually as a result of jumping off furniture. Other fracture sites are related to dog bites (elbow), or restraint (elbow, femur) and horse kicks (femur).

Click here to see a radiograph of a normal humerus in a potbellied pig and compare it to thisradiograph of a distal humeral fracture at the humeral condyle.

Bonus: Here is a normal abdominal-pelvic radiograph of a young potbellied pig. Note the relative small stature of the bones in relation to the heavy body and abdominal contents, so thick they hardly allow x-ray penetration. These are heavy, hard-to-handle animals and it is easy to imagine the stress of even a short fall breaking bones. 

Term
Fractures of the proximal sesamoid bones in horses are often associated with damage to which structure?

A - Suspensory ligament
B - Deep digital flexor tendon
C - Superficial digital flexor tendon
D - Impar ligament
E - Superior check ligament
Definition

Answer: A - suspensory ligament 

The suspensory ligament is most likely to be damaged with fracture of the proximal sesamoids due to its insertion onto these bones. Proximal sesamoid fractures are relatively common, caused by overextension. The prognosis for return to soundness is often predicated on the extent of the damage to the suspensory apparatus. Proximal sesamoid fractures and their attendant suspensory apparatus problems are the number one cause of racetrack deaths.

Click here to see a radiograph of a proximal sesamoid fracture. If you have trouble visualizing the anatomy involved in proximal sesamoid fractures, there are excellent pictures and a clinical summary in Guide to Equine Clinics: LAMENESS vol. II, pp. 116-117, Pasquinis, Jahn & Bahr. 

Term
Which nerve block would most specifically relieve lameness resulting from fracture of the navicular bone?

A - Low four point
B - Tibial and peroneal
C - Abaxial sesamoidean
D - Median and ulnar
E - Palmar digital
Definition

Answer: E - Palmar Digital 

The palmar digital nerve block (also called a PD or heel block) would anesthetize the palmar third of the foot, including the navicular bone. All of the other nerve blocks can desensitize the navicular bone too, but would not be as specific as a palmar digital nerve block.

The abaxial sesamoidean nerve block (ASNB, also called a pastern or foot block) would anesthetize the entire foot and much of the pastern. Sometimes an ASNB is needed to completely eliminate or diagnose navicular lameness, but it is less specific than the PD block.

A low four point nerve block (Also called Low palmar or Volar block) would desensitize the fetlock and areas distal to it. 

The median and ulnar nerve block would anesthetize the carpus and areas distal to it. 

The tibial and peroneal nerve block would anesthetize the tarsus and areas distal to it. 

Term
A 3-year old Quarter horse/Appaloosa cross mare is presented on emergency because she collapsed in the last 30 minutes.

The mare is recumbent and weak, with respiratory stridor, muscle fasiculations, sweating, prolapse of the third eyelid and her lips pulled back spasmodically, like a grin.

T=100.2 F (37.9 C)..[N=99-101.3 F]
HR=24 bpm.............[N=28-40]
RR=16 brpm............[N=10-14]

The horse is hydrated and has had no recent history of colic. During the examination, she improves markedly and within 60 minutes stands and appears almost normal again with no treatment.

A - Hyperkalemic periodic paralysis (HyPP)
B - Myasthenia gravis
C - Equine degenerative myelopathy (EDM)
D - Grass staggers
E - Epilepsy
Definition

Answer: A - Hyperkalemic periodic paralysis 

This is likely to be Hyperkalemic periodic paralysis (HyPP), an autosomal dominant trait found in ~ 4% of Quarter horses.

Heterozygotes often have periodic episodes 15-90 min: prolapse 3rd eyelid, muscle fasiculations, collapse. 25% cases traced back to one stallion named "Impressive".

Onset of signs unpredictable, and many triggers: diets high in POTASSIUM (>1.1%, ie: alfalfa hay, molasses, electrolyte supplements, kelp-based supplements). Fasting, anesthesia, heavy sedation, trailer rides, and stress can precipitate clinical signs.

Grass staggers think "goose-stepping" (overstepping) from ergot on grass. EDM think ataxia in foals 6-8 mos. Epilepsy poorly documented in horses. Myasth. Gravis basically a dog disease.

Term
During the fall, a 5 year-old standardbred mare from northern California is presented with a 5-day history of depression, partial anorexia and undulating fever between 102-106 F (38.9-41.1 C)..[N=99-101.3 F].

Physical exam reveals mucosal petechiae and icterus. The horse has edematous hindlimbs and appears reluctant to move.

Blood drawn for a complete blood count shows inclusion bodies in the neutrophils.

What is the diagnosis?


A - Equine infectious anemia (EIA)
B - Purpura hemorrhagica
C - Equine viral arteritis (EVA)
D - Equine Babesiosis
E - Equine ehrlichiosis
Definition

Answer: E - Equine Erlichiosis 

This is one of the 5 classic "anemia / edema " presentations of horses. (Remember "Big 3 are PEE" Purpura, EIA, EVA; 2 minors are Babesia, ehrlichia (now renamed Anaplasma).

Inclusion bodies in the neutrophils of a California horse with icterus and petechiae and an undulating fever says Equine granulocytic ehrlichiosis (EGE).

Originally classified as Ehrlichia equi, but is NOW called ANAPLASMA phagocytophila due to DNA sequencing studies. EGE is a seasonal necrotizing vasculitis (edema, icterus, petechiae) seen in N. California, suspected to be tick borne.

See dependent edema with Equine viral arteritis (EVA), but also should see respiratory presentation (conjunctivitis/pinkeye, rhinitis) +/- abortions.

Equine infectious anemia (EIA) is rarely seen today because of testing programs, but could present this way. Less likely because 92% cases seen around the Gulf of Mexico states (Texas to Florida), Mississippi river valley AND no inclusion bodies.

Purpura hemorrhagica is a type III Antigen/Antibody/Complement complex disease causing vascultis, petechia, purplish discoloration, usually afebrile, with Hx of recent Strangles (Strep equi equi) or bacterin vaccination.

Equine Babesiosis presents more as a hemolytic anemia: seen in the S. USA, endemic in Southern FLORIDA

Term
Definition

Answer: C - Listeriosis 

This is facial nerve paralysis, most often associated with Listeriosis from silage fed to sheep and goats (cows get it too). Some of the signs are subtle in this image, but classically look for swelling of cheek due to accumulation of food in the buccal space, drooling, ptosis (lowered eyelid) drooping ear, collapsed nostril, flaccid lip, ON AFFECTED SIDE. 

Can see deviated muzzle with lip pulled to OPPOSITE side of lesion. 

Follow this link to see a Merck image of facial nerve paralysis with obvious lip deviation AWAY from affected side in a horse. 

Typically present as encephalitis (depressed, fever) with unilateral limb signs (circling, leaning) and unilateral cranial nerve signs (above). Can see sporadic ABORTIONS with listeriosis at any time in gestation, though more common in 3rd trimester. 

Rx with HIGH DOSES tetracyclines or penicillin. Isolate. Take off silage. +/- IV fluids. 

ZOONOTIC. Usually reportable. Highest risk from handling aborted fetus or necropsy of septicemic animal.

With Polioencephalomalacia, think opisthotonos, headpressing, blind, 'Star-gazing' (dorso-medial strabismus via thiamine deficiency (v. B1)

Caprine Arthritis Encephalitis presents as a polyarthritis in adults, progressive paresis in kids.

Term
[image]
Definition

Answer: E - Clostridium perfringens

These are the hemorrhagic intestines of Clostridium perfringens Type C. Affects piglets up to 3 weeks old with a severe hemorrhagic necrotizing diarrhea. High mortality. 

Brachyspira (Serpulina) hyodysenteriae is a treponemal spirochete that causes Swine Dysentery. Think MUCOID hemorrhagic diarrhea (ie: large intestine) in pigs 6 to 20 weeks old. Diarrheic pigs become dehydrated, profoundly weak, and emaciated.

Lawsonia intracellularis causes Porcine proliferative enteritis, a common diarrheal disease in growing-finishing pigs and younger breeding pigs. 

Clostridium difficile is an emerging diarrhea pathogen principally in neonatal swine- may also see dyspnea, abdominal distention, scrotal edema.

Enterotoxigenic Escherichia coli (ETEC) causes EDEMA disease in recently weaned piglets with peracute death or neurological signs: ataxia, paralysis, and recumbency, and head swelling.

Term
[image]
Definition

Answer: C - infectious laryngotracheitis 

Blood in the trachea and this Hx suggest Infectious Laryngotracheitis (ILT). A highly contagious herpesvirus infection, ILT is characterized by RESPIRATORY signs (rales, severe dyspnea, coughing). In severe forms, mortality can reach 50%, typically due to occlusion of the trachea by blood, mucus or caseous exudates. In most states ILT is REPORTABLE.

Infectious coryza is a milder respiratory disease, with swelling around the eyes and head, sneezing, nasal discharge.

Newcastle disease is characterized primarily by respiratory signs. Severe forms include depression, neurologic signs or diarrhea. Look for GI hemorrhage with most severe form,Viscerotropic Velogenic Newcastle disease (VVND), which is REPORTABLE.

With Fowlpox, only a few birds typically affected with scablike lesions around head. Click here to see Fowlpox.

See only depression before death with Marek's disease . Some birds may have characteristic unilateral paresis (one leg forward, one leg back). On necropsy, see lymphoid tumors in organs and enlarged nerves.

Term
A clinic's waiting room has

A Rottweiler dog with foreleg osteosarcoma
A white mouse with a mammary gland tumor on her belly
A Norway rat with a mammary gland tumor near her neck
A snot-nosed ferret with a chin rash and crusts around the eyes
A thin boa constrictor that can't right itself from dorsal recumbency

Which one of these animals has the BEST prognosis?

A - Ferret
B - Norway rat
C - Boa constrictor
D - White mouse
E - Rottweiler
Definition

Answer: B - the norway rat 

The preferred answer is the rat. MAMMARY GLAND TUMORS are typically BENIGN in rats but MALIGNANT in mice. 

As many as 50% of boid snakes (boas, pythons) harbor the retrovirus which can causeInclusion Body Disease (IBD). IBD can cause regurgitation, weight loss and in later stages neurologic signs, like failure to right itself when snake is placed in dorsal recumbency (on its back). Bad Prognosis- Incurable.

Any ferret with a chin rash and oculonasal discharge has DISTEMPER until proven otherwise (and a grave prognosis). 

A dog with OSTEOSARCOMA has a bad prognosis. Median survival without treatment, or with amputation alone or with palliative radiotherapy alone is only 4 months. Median survival with chemotherapy and surgery (amputation) is still only 10 months. 

Term
A 7 year old female spayed Border Collie is presented with two very goopy, gunk-covered eyes.

A Schirmer tear test finds less than 10 mm/minute of wetting, a decreased result from the normal of at least 15mm / minute of wetting.

The owner reports that the dog has been on "some kind of medicine" for the last 10 days, but it is actually his girlfriend's dog, and he doesn't know what the medicine is.

Keratoconjunctivitis sicca (KCS) secondary to the drug is suspected. Which drug may be causing the KCS?

A - Prednisolone
B - Itraconazole
C - Griseofulvin
D - Trimethoprim sulfa
E - Amitraz
Definition

Answer: D - Trimethoprim Sulfa

Systemic sulfonamides like trimethoprim sulfa have been associated with keratoconjunctivitis sicca (KCS), sometimes irreversibly. Another drug-associated cause of TRANSIENT KCS is the combination of recent general anesthesia and atropine.

Other causes of KCS include: Distemper, Immunologic (think ATOPY), Breed (Pugs, Yorkies), and trauma (Proptosed eyeball).

Term
[image]
Definition

Answer: B - Fenbendazole 

 

Term
A 4-year old quarterhorse mare is presented with a runny left eye and a urine-scalded perineum. No other horses on the farm are sick. Physical exam reveals a corneal ulcer and keratitis OS (left eye), and atrophy of the temporal and masseter muscles.

There is decreased perineal sensation, a weak tail and weak anal sphincter with retained manure. The horse is bright, alert and responsive.
T=102.2 F (39.1 C)..[N=99.0-101.3 F]
HR=40 bpm............[N=28-40]
RR=12 brpm...........[N=10-14]

Which of the following is the most likely diagnosis?br>
NAVLE® Study Strategy seminar Watch it!
Early Registration Sale!

A - Nigropallidal encephalomalacia
B - Equine degenerative myelopathy (EDM)
C - Cauda equina neuritis
D - Equine protozoal myelopathy (EPM)
E - Botulism
Definition

Answer: C - cauda equina neuritis 

Horses with Cauda equina neuritis (also called polyneuritis equi) have a progressive symetric LMN paresis of the tail, bladder, rectum, anal sphincter. Look for urinary incontinence, fecal retention and a weak or paralyzed tail. May see hind limb paresis if lumbosacral spinal cord is affected. 

Cranial nerves can also be affected, but typically cranial involvement is asymetric. May see temporal or masseter atrophy (Cranial Nerve 5), facial paralysis and exposure keratitis (Cranial nerve 7), head tilt or other CNS signs.
Cause is unknown, may be an autoimmune process. Grave prognosis. Eventually euthanized.

Herpesvirus myeloencephalopathy (EHV-1) may also present with urinary incontinence, but this is an uncommon manifestation of equine rhinopneumonitis. You would expect to hear a history of the more common EHV signs in other horses from the same farm, like respiratory disease ("snots") in foals and abortions in mares.

Term
[image]
Definition

Answer: B - Anaplasmosis 

This clinical picture of anemia, icterus and collapse is that of bovine anaplasmosis, a tick-borne obligate intraerythrocytic bacterium in the order Rickettsiales.

Babesiosis has a similar distribution and presentation, but UNlike anaplasmosis, look for hemoglobinuria. Babesia look like little hoops or "pears" inside red cells.

Eperythrozoon is a related blood parasite, but Eperythrozoon looks like this, both inside and outside the red cells.

Supporting users have an ad free experience!